download scanned papers here - Paper 1 Paper II


UPSC PRELIMS 2019 PAPER I -- ANSWER KEYS


विहंगम दृष्टि
  • 2019, कम से कम 2018 जितनी बड़ी चुनौती, अगर उससे बड़ी नहीं तो प्रारंभिक परीक्षा 2019 किसी के भी लिए चुनौतीपूर्ण था - नए या अनुभवी - जिसने वर्ष 2018 की तरह तय पैटर्न की उम्मीद की थी या 2017 की तरह कठिन पैटर्न की, किसी भी चीज एवं सभी चीजों के लिए तैयार होने के बजाय - याद रखें कि ‘प्रमुख प्रतियोगी परीक्षाओं में महत्वपूर्ण ‘अनिश्चितता’ है’। 2017 के पेपर (1 और 2 दोनों) काफी कठिन थे। इससे कटऑफ प्रभावित हुआ। 2018 में, पेपर तथ्यात्मक रूप से कठिन था, हालांकि विश्लेषणात्मक रूप से 2017 जितना खराब नही। 2019 में रुझान जारी रहा, और अधिकांश को पेपर आसान नहीं लगा। सीख (एक बार और) - विशयों की स्थिर तैयारी, गहराई में जाना, कोई यादृच्छिक अनुमान नहीं, विवेक, अभ्यास।

  • प्रमुख परिवर्तनों का सारांश -
    1. कुछ क्षेत्रों में समान रूप से संतुलन -(ए) अंर्तराष्ट्रीय समसामयिकी, राष्ट्रीय समसामयिकी (2018 के 9 एवं 7 की तुलना में 2019 में 6 एवं 7 प्रश्न), (बी) पर्यावरण और पारिस्थितिकी (2018 के 11 की तुलना में इस वर्ष 13 प्रश्न), (सी) भूगोल (2018 में 6 की तुलना में इस वर्ष 5)
    2. कुछ क्षेत्रों में संतुलन बदला - (ए) कृषि सहित विज्ञान और प्रौद्योगिकी (2018 में पूछे गए 14 की तुलना में इस वर्ष 20 प्रश्न), (बी) सरकारी योजनाएं घटकर मात्र 1 रह गईं (पिछले वर्ष की 6 के बजाय), (सी) आधुनिक इतिहास घटकर एक तिहाई (2018 में 13 के स्थान पर इस वर्ष 4), (डी) राजतंत्र में थोड़ी वृद्धि हुई (2018 में पूछे गए 12 के स्थान पर इस वर्ष 15), और (ई) कला और संस्कृति में भारी गिरावट आई (पिछले साल 6 की तुलना में इस वर्ष केवल 1 प्रश्न)
    3. पेपर का कठिनाई स्तर कमोबेश 2018 की तरह ही था तथा ‘रोचकता’वाला हिस्सा भी बरकरार रहा! बहुत अधिक गूढ़ तथ्यों को नहीं पूछा गया था - कुछ सीधे थे
    4. . सूक्ष्म स्तर के आँकड़ों पर बहुत प्रश्न पुछे गए थे (2018 की तरह)
    5. कुछ प्रश्नों में करीबी विकल्पों ने अनुमान लगाना जोखिम भरा कर दिया।
    6. अधिक स्कोर प्राप्त करने के लिए अधिक प्रश्न हल करने के लिए रणनीति असफल हो सकती थी।
    7. केवल लंबी अवधि की तैयारी ही काम आई, ऐन वक्त पर रटने से काम नही चला।
    8. कुल आसान प्रश्नों की संख्या जिन्हें सभी छात्र हल कर सकते थे। सेट डी में, ये प्रश्न 1 (AIIB), 7 (EODB), 14 (बैंकों के एसेट), 17 (वाणिज्यिक पत्र), 22 (PPP), 27 (आरबीआई डेटा स्थानीयकरण), 28 (ईयू जीडीपीआर), 42 (डेनिसोवन), 53 (वैश्विक प्रतिस्पर्धात्मकता रिपोर्ट), 66 (पायरोलिसिस), 70 (माइक्रोबायड्स), 79 (अटल इनोवेशन मिशन) और 84 (ओस की बूंदें) थे। किसी भी छात्र को इन प्रश्नों को छोड़ना नही चाहिए था।
  • सरल प्रश्नों की संख्या कम चूंकि 2016 एवं 2015 में समसामयिक मामलों के प्रश्न अधिक पूछे गए थे, एवं उनमें से कई प्रश्नों को केवल विकल्पों के उन्मूलन द्वारा हल किया जा सकता था, एवं कई प्रश्न इतने सरल थे कि यदि आपने उससे संबधित अवधारणा को केवल एक बार पढ़ा हो तो भी हल कर सकते थे, 2017 जोर का झटका था। 2018 ने परंपरा को जारी रखा। और 2019 भी अलग नहीं था। यदि आप इस तरह के पेपर के लिए तैयार थे, तो कुल मिलाकर मदद मिली होगी। सीख - यह एक सापेक्ष प्रदर्शन परीक्षण है, निरपेक्ष नहीं।

  • टॉपिक वाइज बैलेंस बदल गया 2019 के पेपर में 2017 एवं 2018 के विपरीत केवल एक क्षेत्र के प्रश्नों का बोलबाला था, इसमें राजनीति से 22 प्रश्न एवं सरकारी योजनाओं से 12 प्रश्न पुछे गए। इस वर्ष सबसे प्रमुख विषय थे - विज्ञान और प्रौद्योगिकी, कृषि को मिलाकर (20 प्रश्न), 15 भारतीय अर्थव्यवस्था से, 15 राजनीति से और 13 पर्यावरण और पारिस्थितिकी से। सीख - कुछ भी उम्मीद न करें

  • हमारा समर्थन - पीटी आईएएस अकादमी में, हमने प्रत्येक विषय पर यथासंभव प्रयास करने की कोशिश की है। हम अपने छात्रों को परिणामों में बहुत शुभकामनाएं देते हैं!

  • सरल प्रश्नों की संख्या सीमित- ऊपर वर्णित 13 प्रश्न आसान थे।

  • सरकारी योजनाओं पर प्रश्न सीमित - 2017 के विपरीत बहुत अधिक प्रश्न नहीं थे।

  • अस्पष्ट उत्तर - 2018 एवं 2017 के विपरीत, कोई बड़ी अस्पष्टता नहीं थी, लेकिन व्याख्या के कुछ संभावित मुद्दे थे। वर्ष 2017 में बटरफ्लाई /वोट /आईवीसी/जिप्सम आदि पर सवाल थे। यह अच्छी खबर है।

  • इस स्तर पर, हम छात्रों को प्रीलिम्स के लिए ‘कट-ऑफ’ के बारे में किसी भी अटकलों से बचने के लिए सुझाव देना चाहेंगे। एक व्यापक दिशानिर्देश सामान्य श्रेणी - 95 से 103 मार्क्स, ओबीसी - लगभग समान और एससी/एसटी - 85 से 92 अंक हो सकता है। यदि आप कहीं भी इन के पास हैं, तो मुख्य परीक्षा की तैयारी करें। निराश ना हो, आशा ना खोएं। शुभकामनाएं!


विषयवार वर्गीकरण



विस्तृत प्रश्नवार समाधान (Set D)

1. एशियाई आधारिक-संरचना निवेश बैंक {एशियन इंफ्रास्ट्रक्चर इंवेस्टमे।ट बैंक (AIIB)} के संदर्भ में, निम्नलिखित कथनों पर विचार कीजिए :

  1. AIIB के 80 से अधिक सदस्य राष्ट्र हैं।
  2. AIIB में भारत सबसे बड़ा शेयरधारक है।
  3. AIIB में एशिया से बाहर का कोई सदस्य नहीं हैं।

उपर्युक्त में से कौन-सा/से कथन सही है/हैं?

  1. केवल 1
  2. केवल 2 और 3
  3. केवल 1 और 3
  4. 1, 2 और 3

उत्तर (ए) केवल कथन 1 सही है। एशियन इन्फ्रास्ट्रक्चर इन्वेस्टमेंट बैंक (एआईआईबी), एशिया-प्रशांत क्षेत्र में बुनियादी ढांचे के निर्माण में मदद करने के लिए एक बहुपक्षीय विकास बैंक है जो चीन की पहल है। इसका मुख्यालय बीजिंग में है तथा इसकी शुरूआत जनवरी 2016 में हुई। इसके अनुमोदित सदस्यों की संख्या अब बढ़कर 97 तक पहुंच गई है जिसमें एशिया के अलावा बाकि विश्व के देश भी शामिल हैं। भारत की इसमें हिस्सेधारी सर्वाधिक नही है, जबकि चीन की है। स्पष्ट रूप से, कथन 2 एवं 3 सही नही हैं। अतः विकल्प (ए) सही उत्तर है।



2. हाल ही में, भारतीय बैंको और वित्तीय संस्थाओं द्वारा अंतर-ऋणदाता करार (इंटर-क्रेडिटर ऐग्रीमेंट) पर हस्ताक्षर करने का क्या उद्देष्य था?

  1. भारत सरकार के राजकोशीय घाटे और चालू लेखा घाटे के वर्षानुवर्ष पड़ने वाले भार को कम करना
  2. केन्द्रीय और राज्य सरकारों की आधारिक-संरचना परियोजनाओं को संबल प्रदान करना
  3. रू. 50 करोड़ या अधिक के ऋणों के आवेदनों के मामले में स्वतंत्र नियामक के रूप में कार्य करना
  4. रू. 50 करोड़ या अधिक की दबावयुक्त परिसंपत्तियों (स्ट्रेस्ड ऐसेट्स) का, जो सह-संघ उधारी (कॉन्सॉर्टियम लेंडिंग) के अंतर्गत हैं, अधिक तेजी से समाधान करने का लक्ष्य रखना।

उत्तर (डी) पहला विकल्प अप्रासंगिक है क्योंकि भारतीय बैंकों एवं वित्तीय संस्थानों द्वारा इस तरह की कोई कोशिश नहीं की जा सकती है। वे इस तरह राजकोषीय एवं मौजूदा घाटे को प्रभावित नहीं कर सकते हैं! विकल्प (बी) सही लगता है, लेकिन तथ्यात्मक रूप से सही नही है। विकल्प (सी) पूरी तरह से सही नही है। विकल्प (डी) सही उत्तर है। इंटर-क्रेडिटर समझौता (आईसीए) भारतीय बैंक संघ द्वारा तैयार किया गया था एवं स्ट्रेस्ड असेट्स रिसोल्युशन पर सुनील मेहता समिति की सिफारिशों का पालन करता है। इस पर प्रारंभ में, भारतीय स्टेट बैंक, बैंक ऑफ इंडिया एवं कॉर्पोरेशन बैंक सहित 24 उधारदाताओं ने हस्ताक्षर किए, एवं कुछ बैंक बाद में शामिल हुए। संदर्भ छवि यहां है।


3. सार्वजनिक क्षेत्रक बैंकों के अध्यक्षों का चयन कौन करता है?

  1. बैंक बोर्ड ब्यूरो
  2. भारतीय रिज़र्व बैंक
  3. केंद्रीय वित्त मंत्रालय
  4. संबंधित बैंक का प्रबंध

उत्तर (ए) बैंक बोर्ड ब्यूरो के अस्तित्व की शुरूआत 2014 में भारत में बैंकों के बोर्ड के नियमन के लिए बनाई गई समिति की सिफारिशों से होती है। ब्यूरो ने 01 अप्रैल, 2016 से एक स्वायत्त अनुशंसात्मक निकाय के रूप में कार्य करना शुरू किया, यह भारत के सार्वजनिक क्षेत्र के बैंकों के संभावित अध्यक्षों के लिए नामों का चयन करता है। श्री भानु प्रताप शर्मा को 2018 में पूर्व कैग प्रमुख विनोद राय की जगह दूसरा अध्यक्ष नियुक्त किया गया। विकल्प (बी) सही नही है, क्योंकि आरबीआई नियामक है। विकल्प (सी) संभावित रूप से सही लगता है, लेकिन अंतिम नियुक्ति केंद्रीय मंत्रिमंडल की नियुक्ति समिति द्वारा की जाती है, वह विकल्प नहीं दिया गया है, इसलिए हम (ए) के साथ जाएंगे। विकल्प (डी) स्पष्ट रूप से सपनों की दुनिया से है!


4. निम्नलिखित कथनों पर विचार कीजिएः

  1. पेट्रोलियम और प्राकृतिक गैस नियामक बोर्ड {पेट्रोलियम ऐंड नेचुरल गैस रेग्युलेटरी बोर्ड (PNGRB)}भारत सरकार द्वारा स्थापित प्रथम नियामक निकाय है।
  2. PNGRB का एक कार्य गेस के लिए प्रतियोगी बाजारों को सुनिश्चित करना है।
  3. PNGRB के निर्णयों के विरूद्ध अपील, विद्युत अपील अधिकरण के समक्ष की जाती है।

उपर्युक्त में से कौन-से कथन सही हैं?

  1. केवल 1 और 2
  2. केवल 2 और 3
  3. केवल 1 और 3
  4. 1, 2 और 3

उत्तर (बी) पेट्रोलियम एवं प्राकृतिक गैस नियामक बोर्ड (पीएनजीआरबी) का गठन पेट्रोलियम एवं प्राकृतिक गैस विनियामक बोर्ड अधिनियम, 2006 (2006 की संख्या 19) के तहत 31 मार्च, 2006 को अधिसूचित राजपत्र अधिसूचना के माध्यम से किया गया था। इसलिए, कथन 1 सही नही है, क्योंकि यह निश्चित रूप से भारत सरकार द्वारा स्थापित ‘पहला नियामक निकाय’ नहीं है (यह कथन किस क्षेत्र से है, यह नही दिया गया है इसलिए हम सभी क्षेत्रों को मान कर चलते हैं)। यदि कथन 1 सही नही है, तो विकल्प (ए), (सी) एवं (डी) सही नही हैं। अतः, केवल (बी) सही है। कथन 2 एवं 3 वास्तव में सही हैं। विद्युत अधिनियम, 2003 (2003 की संख्या 36) की धारा 110 के तहत स्थापित अपीलीय न्यायाधिकरण पीएनजीआरबी द्वारा जारी आदेश के तहत अपीलीय न्यायाधिकरण है। संदर्भ छवि यहां है।


5. संचार प्रौद्योगिकियों के संदर्भ में, LTE (लॉन्ग-टर्म इवॉल्यूशन) और VoLTE(वॉइस ओवर लॉन्ग-टर्म इवॉल्यूशन) के बीच क्या अंतर है/हैं?

  1. LTE को साधारणतः 3G के रूप में विपणित किया जाता है तथा VoLTE को साधारणतः उन्नत 3G के रूप में विपणित किया जाता है।
  2. LTE डैटा-ओन्लि तकनीक है और VoLTE वॉइस-ओन्लि तकनीक है।

नीचे दिए गए कूट का प्रयोग कर सही उत्तर चुनिए।

  1. केवल 1
  2. केवल 2
  3. 1 और 2 दोनों
  4. न तो 1, न ही 2

उत्तर (डी) एलटीई शब्द का अर्थ है, दीर्घकालिक विकास तथा यह वायरलेस डेटा संचार प्रौद्योगिकी एवं जीएसएम/यूएमटीएस मानकों के विकास के लिए ईटीएसआई (यूरोपीय दूरसंचार मानक संस्थान) के स्वामित्व वाला एक पंजीकृत ट्रेडमार्क है। एलटीई का प्रायः ‘4जी एलटीई एवं एडवांस 4जी’ के रूप में विपणन किया जाता है, इसलिए कथन 1 सही नही है। तो विकल्प (ए) एवं (सी) सही नही हैं। एलटीई को आमतौर पर 3.95जी के रूप में भी जाना जाता है, एवं इसे मुख्य रूप से डेटा-ओनली तकनीक के लिए डिजाइन किया गया था। वॉइस ओवर लॉन्ग-टर्म इवोल्यूशन (वीओएलटीई), आईओटी डिवाइसेस एवं वीयरेबल्स के साथ मोबाइल फोन एवं डेटा टर्मिनलों के लिए हाई-स्पीड वायरलेस कम्युनिकेशन के लिए एक मानक है। वीओएलटीई में 3 जी यूएमटीएस की तुलना से तीन गुना एवं 2 जी जीएसएम की तुलना से छह गुना तक अधिक आवाज एवं डेटा क्षमता है। यह आवाज एवं डेटा, दोनों को एक साथ ले जाता है। इसलिए, कथन 2 भी सही नही है। इसलिए, विकल्प (डी) हमारी पसंद है। संदर्भ छवि यहां है।


6. मातृत्व लाभ (संशोधन) अधिनियम, 2017 के संबंध में, निम्नलिखित में से कौन-सा/से कथन सही है/हैं?

  1. गर्भवती महिलाएँ, प्रसव-पूर्व तीन महीने और प्रसवोत्तर तीन महीनें के लिए सवेतन अवकाश की हकदार हैं।
  2. शिशुगृहों वाले प्रतिष्ठानों के लिए माता को प्रतिदिन कम-से-कम छह बार शिशुगृह जाने की अनुमति देना अनिवार्य होगा।
  3. दो बच्चों वाली महिलाओं को न्यूनीकृत हक मिलेंगे।

नीचे दिये गए कूट का प्रयोग कर सही उत्तर चुनिए ।

  1. केवल 1 और 2
  2. केवल 2
  3. केवल 3
  4. 1, 2 और 3

उत्तर (सी) कथन 2 सही नही है। क्योकि नियम के अनुसार, माँ दिन में 6 बार नहीं बल्कि दिन में 4 बार शिशुगृह (क्रेश) में जाने करने की हकदार है। अतः, विकल्प (ए), (बी) एवं (डी) सही नही हैं! तो, केवल (सी) सही है। यदि आप जानते हैं कि 2 सही नही है, तो पूरा प्रश्न तुरंत हल हो जाएगा। मातृत्व लाभ अधिनियम (एमबीए), 1961 के (1 अप्रैल, 2017 से प्रभावी) के 2017 संशोधन में कहा गया है कि - (ए) महिला कर्मचारियों के लिए सवेतन मातृत्व अवकाश 12 सप्ताह से बढ़ाकर 26 सप्ताह कर दिया गया है, जब तक कि उनके दो या अधिक जीवित बच्चे न हों। (बी) पहली बार गोद लेने वाली मां एवं एक कमीशन्ड माँ (सरोगेसी के तहत बच्चे को जन्म देने वाली माँ) के अधिकारों को मान्यता दी गई है, जो 12 सप्ताह तक के सवेतन मातृत्व अवकाश का दावा कर सकती है। (सी) मातृत्व अवकाश के समाप्त होने के बाद ‘घर से काम’ करने का विकल्प लाभदायक हो सकता है। (डी) एवं, 50 या उससे अधिक कर्मचारियों वाले प्रत्येक प्रतिष्ठान के लिए 1 जुलाई, 2017 से अनिवार्यतः शिशुगृह की सुविधा (डे केयर), जिसमें माताओं का प्रति दिन चार बार जाने का अधिकार भी शामिल है। ऊपरोक्त (ए) से स्पष्ट है कि कथन (3) सही है।


7. निम्नलिखित में से कौन-सा एक विश्व बैंक के ’कारोबार सुगमता सूचकांक (ईज़ ऑफ डूइंग बिजनेस इंडेक्स)’ का उप-सूचकांक नहीं है?

  1. कानून और व्यवस्था बनाए रखना
  2. करों का भुगतान करना
  3. संपत्ति का पंजीकरण कराना
  4. निर्माण परमिट संबंधी कार्य करना

उत्तर (ए) यह एक प्रश्न था, जिसके आने की उम्मीद भी थी क्योंकि क्योंकि ईज ऑफ डूइंग बिजनेस (व्यापार सुगमता) एक ज्वलंत विषय है! विश्व बैंक द्वारा संचालित सूचकांक में निम्नलिखित शामिल हैं - व्यवसाय शुरू करना, निर्माण परमिट से निपटना, बिजली प्राप्त करना, संपत्ति नामांकन करवाना, ऋण प्राप्त करना, अल्पसंख्यक निवेशकों की सुरक्षा करना, करों का भुगतान करना, सीमाओं के पार व्यापार करना, अनुबंधों को लागू करना एवं शोध-अक्षमता का समाधान करना। अतः विकल्प (ए), क्योंकि कानून एवं व्यवस्था का रखरखाव शामिल नहीं है। आप इसे देखकर यह अनुमान लगाने की कोशिश कर सकते थे कि विकल्प (ए) व्यापार के मुद्दे की तुलना में आंतरिक सामाजिक-राजनीतिक मुद्दे के साथ अधिक जुड़ा है। संदर्भ छवि यहां है।


8. भारत में निम्नलिखित में से किसमें एक महत्वपूर्ण विशेषता के रूप में ’विस्तारित उत्पादक दायित्व’ आरंभ किया गया था?

  1. जैव चिकित्सा अपशिष्ट (प्रबंधन और हस्तन) नियम, 1998
  2. पुनर्चक्रित प्लास्टिक (निर्माण और उपयोग) नियम, 1999
  3. ई-अपशिष्ट (प्रबंधन और हस्तन) नियम, 2011
  4. खाद्य सुरक्षा और मानक विनियम, 2011

उत्तर (सी) यह प्रश्न काफी मुश्किल था, क्योंकि हो सकता हो (‘प्लास्टिक’ के कारण) आपने विकल्प (बी) को स्वाभाविक रूप से सही समझा हो। लेकिन दिए गए विकल्पों में से, ईपीआर अवधारणा पहली बार ई-कचरा (प्रबंधन एवं हैंडलिंग) नियम, 2011 में आई थी, जिसमें कहा गया था कि ‘(1) ‘’एक्सटेंडेट प्रोड्यूसर रिस्पाँसिबिलिटी’’ का अर्थ, किसी बिजली या इलेक्ट्रॉनिक उपकरणों के निर्माता की जिम्मेदारी केवल विनिर्माण से परे उनके उत्पाद के समाप्त हो जाने पर उनके ऊचित पर्यावरण हिताय प्रबंधन तक, है। प्लास्टिक के लिए ईपीआर की अवधारणा पहली बार प्लास्टिक अपशिष्ट (प्रबंधन एवं हैंडलिंग) नियम, 2011 में आई थी, न कि 1999 में।


9. भारतीय खाद्य निगम के लिए खाद्यान्नों की आर्थिक लागत में न्यूनतम समर्थन मूल्य और किसानों को भुगतान किए गए बोनस (यदि कुछ है) के साथ-साथ और क्या शामिल है/हैं?

  1. केवल परिवहन लागत
  2. केवल ब्याज लागत
  3. प्रापण प्रासंगिक प्रभार तथा वितरण लागत
  4. प्रापण प्रासंगिक प्रभार तथा गोदामों के प्रभार

उत्तर (सी) एक मुश्किल प्रश्न। एफसीआई (भारतीय खाद्य निगम) आर्थिक लागत को अधिग्रहण लागत एवं वितरण लागत के योग के रूप में परिभाषित करता है। नजदीकी विकल्प (सी) है। संदर्भ छवि यहां है।


10. किसी भी देश के संदर्भ में, निम्नलिखित में से किसे उस देश की सामाजिक पूँजी (सोशल कैपिटल) के भाग के रूपा में समझा जाएगा?

  1. जनसंख्या में साक्षरों का अनुपात
  2. इसके भवानों, अन्य आधारिक संरचना और मशीनों का स्टॉक
  3. कार्यशील आयु समूह में जनसंख्या का आमाप
  4. समाज में आपसी भरोसे और सामंजस्य का स्तर

उत्तर (डी) विकल्प (बी) एवं (डी) को सीधे खारिज किया जा सकता है। अब विकल्प (ए) एवं (डी) के बीच में, विकल्प (ए) को बहुत ही संकीर्ण रूप से परिभाषित किया गया है। इसलिए, (डी) को चुना जाना चाहिए। सामाजिक पूंजी, सामाजिक विज्ञान की एक अवधारणा है जिसमें लाभों को सुरक्षित करने के लिए व्यक्तियों की क्षमता शामिल तथा जिसमें सामाजिक नेटवर्क में सदस्यता के माध्यम से समस्याओं के समाधान खोजे जाते हैं।


download scanned papers here - Paper 1 Paper II


11. सेवा क्षेत्र उपागम किसके कार्यक्षेत्र के अधीन कार्यान्वित किया गया था?

  1. एकीकृत ग्रामीण विकास कार्यक्रम
  2. अग्रणी बैंक योजना (लीड बैंक स्कीम)
  3. महात्मा गाँधी राष्ट्रीय ग्रामीण रोजगार गारंटी योजना
  4. राष्ट्रीय कौशल विकास मिशन

उत्तर (बी) सर्विस एरिया अप्रोच (एसएए), लीड बैंक योजना (एलबीएस) की एरिया अप्रोच (एए) का एक उन्नत संस्करण है। एसएए योजना के तहत ग्रामीण एवं अर्ध-शहरी क्षेत्र में प्रत्येक वाणिज्यिक बैंक/क्षेत्रीय ग्रामीण बैंक (आरआरबी) को नियोजित एवं क्रमिक विकास के लिए 15 से 25 गांवों की सेवा के लिए नामित किया जाता है। इस योजना को 1989 में शुरू किया गया था तथा दिसंबर 2004 में इसकी समीक्षा की गई। लीड बैंक योजना 1969 में शुरू की गई थी, जिसमें बैंकों (सार्वजनिक एवं निजी) को उनके आवंटित जिलों के लिए निभाई जाने वाली प्रमुख भूमिकाओं को निश्चित करने की परिकल्पना थी। ऐसा बैंक जिसके पास एक दिए गए जिले के ग्रामीण क्षेत्रों में शाखाओं का अपेक्षाकृत बड़ा नेटवर्क है जो पर्याप्त वित्तीय एवं जनशक्ति संसाधनों से संपन्न है, को यह जिम्मेदारी दी जाती है।


12. भारत में गौण खनिज के प्रबंधन के संदर्भ में, निम्नलिखित कथनों पर विचार कीजिए :

  1. इस देश में विद्यमान विधि के अनुसार रेत एक ’गौण खनिज’ है।
  2. गौण खनिजों के खनन पट्टे प्रदान करने की शक्ति राज्य सरकारों के पास है, किन्तु गौण खनिजों को प्रदान करने से संबंधित नियमों को बनाने के बारे में शक्तियाँ केन्द्र सरकार के पास हैं।
  3. गौण खनिजों के अवैध खनन को रोकने के लिए नियम बनाने की शक्ति राज्य सरकारों के पास है। उपर्युक्त में से कौन-सा/से कथन सही है/हैं?
  1. केवल 1 और 3
  2. केवल 2 और 3
  3. केवल 3
  4. 1, 2 और 3

उत्तर (ए) विकल्पों को देखें - कथन 3 चारों में है, इसलिए यह सही है। यदि आप जानते हैं कि रेत एक ‘माईनर मिनरल’ (लघु खनिज) है, तो विकल्प (बी) एवं (सी) भी खारिज किए जा सकते हैं। हमें अब विकल्पों (ए) एवं (डी) की जाँच करनी होगी। भारत 88 खनिजों का उत्पादन करता है जिसमें 55 ‘माईनर मिनरल’ शामिल हैं। खान एवं खनिज (विकास एवं विनियमन) अधिनियम, 1957 की धारा 3 (ई) के अनुसार, ‘माईनर मिनरल’ का अर्थ है पत्थर, बजरी, साधारण मिट्टी, निर्धारित उद्देश्यों के लिए उपयोग किए जाने वाले साधारण रेत के अलावा अन्य किसी भी खनिज का निर्माण करना, जो मध्य सरकार, सरकारी राजपत्र में अधिसूचना द्वारा, एक मामूली खनिज होने की घोषणा कर सकती है। ‘खनिज’ शब्द में खनिज तेल- प्राकृतिक गैस एवं पेट्रोलियम को छोड़कर सभी खनिज शामिल हैं। केंद्र सरकार के पास एमएमडीआर अधिनियम, 1957 की धारा 3 (ई) के तहत ‘लघु खनिजों’ को अधिसूचित करने की शक्ति है। दूसरी ओर, एमएमडीआर अधिनियम, 1957 के धारा 15 के अनुसार में राज्य सरकारों के पास ‘माईनर मिनरल’ की खनन के संबंध में रियायतें देने एवं मामूली खनिजों पर रॉयल्टी वसूलने के लिए नियम बनाने की पूरी शक्तियां हैं। इसलिए, कथन 2 सही नही है। इसलिए, (ए) सही विकल्प है। संदर्भ छवि यहां है।


13. निम्नलिखित कथनों पर विचार कीजिए :

  1. भारत का अधिकांश विदेशी ऋण सरकारी सत्वों के ऋणी होने के द्वारा है।
  2. भारत का सारा विदेशी ऋण US डॉलर मूल्यवर्ग में है।

उपर्युक्त में से कौन-सा/से कथन सही है/हैं?

  1. केवल 1
  2. केवल 2
  3. 1 और 2 दोनों
  4. न तो 1, और न ही 2

उत्तर (डी) यह एक बहुत अच्छा प्रश्न है! कथन 1 सही नही है, क्योंकि अधिकांश विदेशी ऋण, निजी संस्थाएँ लेती हैं। विदेशी ऋण वह धन है जो किसी देश में उधारकर्ताओं द्वारा विदेशी ऋणदाताओं से लिया जाता है। सितंबर 2018 के अंत में भारत का विदेशी ऋण 510.4 बिलियन डॉलर था, इसमें से अधिकांश (डॉलर 407.7 बिलीयन) निजी व्यवसायों द्वारा बकाया था, जो विदेशी उधारदाताओं से आकर्षक दरों पर उधार लेता है। कथन 2 भी सही नही है, क्योंकि भारत के बाहरी ऋण को यूरो, एसडीआर, जापानी येन एवं पाउंड स्टर्लिंग के अलावा अमेरिकी डॉलर में दर्शाया जाता है। मार्च 2018 के अंत में, 49.5 प्रतिशत की हिस्सेदारी के साथ अमेरिकी डॉलर का ऋण भारत के विदेशी ऋण का सबसे बड़ा घटक था, इसके बाद भारतीय रुपया (35.8 प्रतिशत), एसडीआर (5.5 प्रतिशत), जापानी येन (4.8 प्रतिशत) था। ) एवं यूरो (3.4 प्रतिशत) आदि आते हैं। संदर्भ छवि यहां है।


14.भारत में किसी वाणिज्यिक बैंक की परिसंपत्ति में निम्नलिखित में से क्या शामिल नहीं है?

  1. अग्रिम
  2. जमा
  3. निवेश
  4. माँग तथा अल्प सूचना मुद्रा (मनी ऐट कॉल ऐंड शॉर्ट नोटिस)

उत्तर (बी) मूल तत्व यह है कि जमा राशि का भुगतान किया जाना होता है, एवं इसलिए वह एक दायित्व होगा, व ना कि परिसंपत्ति। भारतीय वाणिज्यिक बैंकों की प्रमुख संपत्तियाँ हैं - पास की नकद रकम, कॉल एवं छोटी सूचना पर धन, निवेश, ऋण, अग्रिम, एवं बिलों में छूट एवं खरीदी। जबकि प्रमुख दायित्वों में पेड-अप कैपिटल एवं रिजर्व, डिपॉजिट, उधार आदि शामिल हैं, यह एक सरल प्रश्न था!


15. भारत के संदर्भ में, मुद्रा संकट के जोखिम को कम करने में निम्नलिखित में से किस/किन कारक/कारकों का योगदान है?

  1. भार के IT सेक्टर के विदेशी मुद्रा अर्जन का
  2. सरकारी व्यय के बढ़ने का
  3. विदेशस्थ भारतीयों द्वारा भेजे गए धन का

नीचे दिए गए कूट का प्रयोग कर सही उत्तर चुनिए।

  1. केवल 1
  2. केवल 2 और 3
  3. केवल 2
  4. 1, 2 और 3

उत्तर (बी) ‘मुद्रा संकट’ एक ऐसी स्थिति है जब भारतीय रुपया अचानक मूल्य खोने लगता है, एवं तेजी से मूल्यह्रास करता है। यह विदेशी खाते पर दबाव डालता है। यह सामान्यतः तब होता है जब अधिक डॉलर सिस्टम से बाहर निकलता है । इसलिए ऐसा कुछ भी जो डॉलर को ला सके वह मुद्रा संकट के जोखिम को कम करने में मदद करता है। भारतीय आईटी क्षेत्र की आय का अदृश्य खाते के माध्यम से डॉलर लाने में प्रत्यक्ष योगदान है, इसलिए कथन 1 सही है। विदेश से भेजे गए डॉलर का हमेंशा स्वागत हैं, एवं भारत इसका नंबर 1 प्राप्तकर्ता है, इसलिए कथन 3 सही है। कथन 2 को प्रश्न के साथ सीधे-सीधे कोई संबंध नहीं है। संदर्भ छवि यहां है।


16. निम्नलिखित में से किस एक का का यह सुझाव था कि राज्यपाल को उस राज्य के बाहर का एक प्रतिष्ठित व्यक्ति होना चाहिए और उसे एक ऐसा तटस्थ व्यक्ति होना चाहिए जिसके गहन राजनीतिक जुड़ाव न हों या उसने हाल के पिछले वर्षों में राजनीति में भाग नहीं लिया हो?

  1. पहला प्रशासनिक सुधार आयोग (1966)
  2. राजमन्नार समिति (1969)
  3. सरकारिया आयोग (1983)
  4. संविधान के कार्यचालन की समीक्षा हेतु राष्ट्रीय आयोग (2000)

उत्तर (सी) सरकारिया आयोग ने इसकी सिफारिश की थी। इसमें कहा गया है कि - 1. एक राज्यपाल की योग्यता पर - उसे जीवन के कुछ क्षेत्रों में प्रतिष्ठित होना चाहिए, उसे राज्य के बाहर से एक व्यक्ति होना चाहिए, उसे गहन राजनीतिक संबंधों से अलग होना चाहिए, या उसने हाल के दिनों में राजनीति में भाग ना लिया हो, उन्हें सत्ता पक्ष का सदस्य नहीं होना चाहिए, आदि।




17.पंजीकृत विदेशी पोर्टफोलियो निवेशकों द्वारा उन विदेशी निवेशकों को, जो स्वयं को सीधे पंजीकृत कराए बिना भारतीय स्टॉक बाजार का हिस्सा बनना चाहते हैं, निम्नलिखित में से क्या जारी किया जाता है ?

  1. जमा प्रमाण- पत्र
  2. वाणिज्यिक पत्र
  3. वचन-पत्र (प्रामिसरी नोट)
  4. सहभागिता पत्र (पार्टिसिपेटरी नोट)

उत्तर (डी) यह विषय हाल में खबरों में रहा है। हम सभी का विश्लेषण करते हैं - 1. जमा राशि का प्रमाण पत्र (सीडी) एक परक्राम्य मुद्रा बाजार साधन है एवं यह एक निर्दिष्ट समय अवधि के लिए बैंक या अन्य पात्र वित्तीय संस्थान में जमा किए गए धन के खिलाफ एक प्रमोटर नोट के रूप में जारी किया जाता है। यह भारत में 1989 से शुरू किया गया था। 2. वाणिज्यिक पत्र (सीपी) एक अनसिक्योर्ड मनी मार्केट इंस्ट्रूमेंट है जिसे एक वचन पत्र के रूप में जारी किया जाता है। 3. एक वचन पत्र का अर्थ है एक हस्ताक्षरित दस्तावेज जिसमें एक निर्दिष्ट तिथि या मांग पर किसी निर्दिष्ट व्यक्ति को एक निर्दिष्ट राशि का भुगतान करने का लिखित वादा किया गया हो। 4. पार्टिसिपेटरी नोट्स (पी-नोट्स या पीएन) भारतीय प्रतिभूतियों एवं भारतीय प्रतिभूति एवं विनिमय बोर्ड (सेबी) के साथ पंजीकृत किए बिना भारतीय प्रतिभूतियों में निवेश करने के लिए हेज फंड द्वारा आवश्यक वित्तीय साधन हैं। पी-नोट्स ऑफशोर डेरिवेटिव इन्वेस्टमेंट्स (ओडीआई) माने जाने वाले निवेशों के समूह में से हैं। संदर्भ छवि यहां है।


18. निम्नलिखित कथनों पर विचार कीजिए :

  1. विधि के अनुसार, प्रतिपूरक वनीकरण कोष प्रबंधन एवं योजना प्राधिकरण, राष्ट्रीय तथा राज्य, दोनों स्तरों पर होते हैं।
  2. प्रतिपूरक वनीकरण निधि अधिनियम, 2016 के अधीन चलाए गए प्रतिपूरक वनीकरण कार्यक्रमों में लोगों की सहभागिता अनिवार्य (मैंडेटरि) है।

उपर्युक्त में से कौन-सा/से कथन सही है/हैं?

  1. केवल 1
  2. केवल 2
  3. 1 और 2 दोनों
  4. न तो 1, न ही 2

उत्तर (ए) कथन 1 सही है। क्षतिपूरक वनीकरण कोष अधिनियम, 2016, अध्याय 3, धारा 8, राष्ट्रीय प्राधिकरण के बारे में तथा धारा 9 राज्य प्राधिकरण के बारे में बताती है। कथन 2 सही नही है। जैसा कि अधिनियम में, इसके तहत किए गए कार्यक्रमों में लोगों की अनिवार्य भागीदारी का कोई उल्लेख नहीं है। मुख्य शब्द ‘अनिवार्य’ था। सर्वोच्च न्यायालय ने जनवरी 2019 में निर्देश दिया कि क्षतिपूरक वनीकरण कोष प्रबंधन एवं योजना प्राधिकरण (कैंपा) के तहत लगभग 54,000 करोड़ रुपये के सभी फंड केंद्र सरकार द्वारा बनाए गए एक नए प्राधिकरण को हस्तांतरित कर दिए जाएं, जो केंद्र एवं राज्य सरकार दोने के लिए उपयोगी हो। संदर्भ छवि यहां है।


19. भारत में दूरसंचार, बीमा, विद्युत आदि जैसे क्षेत्रकों में स्वतंत्र नियामकों का पुनरीक्षण निम्नलिखित में से कोन करते/करती हैं?

  1. संसद द्वारा गठित तदर्थ समितियाँ
  2. संसदीय विभाग संबंधी स्थायी समितियाँ
  3. वित्त आयोग
  4. वित्तीय क्षेत्र विधायी सुधार आयोग
  5. नीति (NITI) आयोग

नीचे दिए गए कूट का प्रयोग कर सही उत्तर चुनिए।

  1. 1 और 2
  2. 1, 3 और 4
  3. 3, 4, और 5
  4. 2 और 5

उत्तर (ए) कथनों को बारिकी से देखें। क्या नीति आयोग (एक गैर-वैधानिक, गैर-संवैधानिक निकाय) को इतनी बड़ी जिम्मेदारी दी जा सकती है?, नही। इसलिए, 5 उत्तर का हिस्सा नहीं हो सकता। उसी तरह, वित्त आयोग का नियामकों से कोई लेना-देना नहीं है! इसलिए, 3 को भी खारिज किया जा सकता है। अंततः (ए) को छोड़कर सभी विकल्प खारिज हो जाते हैं। भारत में, संसद निम्नलिखित साधनों से नियामकों की जांच करती है - (1) प्रश्नकाल (2) संसद में चर्चा एवं (3) संसदीय समितियाँ (विभाग संबंधित स्थायी समितियाँ, वित्त समितियाँ जैसे कि प्राक्कलन समिति, लोक लेखा समिति (पीएसी), एवं तदर्थ समितियाँ)। विधायी निरीक्षण के अन्य साधन, जैसे कि नियामकों द्वारा संसद मंे वार्षिक रिपोर्ट प्रस्तुत करना, कभी-कभी अधिनियमितियों में लिखे जाते हैं या फिर संसद की प्रक्रियाओं के नियमों में निर्धारित किए जाते हैं।


20. भारत की पंचवर्षीय योजनाओं के संदर्भ में, निम्नलिखित में से कौन-सा/से कथन सही है/हैं?

  1. दूसरी पंचवर्षीय योजना से बुनियादी तथा पूँजीगत वस्तु उद्योगों के प्रतिस्थापन की दिशा में निश्चयात्मक जोर दिया गया।
  2. चौथी पंचवर्षीय योजना में संपत्ति तथा आर्थिक शक्ति को बढ़ते संकेंद्रण की पूर्व प्रवृत्ति के सुधार का उद्देश्य अपनाया गया।
  3. पाँचवीं पंचवर्षीय योजना में, पहली बार, वित्तीय क्षेत्रक को योजना के अभिन्न अंग के रूप में शामिल किया गया।

नीचे दिए गए कूट का प्रयोग कर सही उत्तर चुनिए ।

  1. केवल 1 और 2
  2. केवल 2
  3. केवल 3
  4. 1, 2 और 3

उत्तर (ए) सभी तीनों सही हैं। कथन 1 का हम यह अर्थ निकाल सकते हैं कि भारत अपने मूलभूत एवं मुख्य वस्तुओं के उद्योगों के लिए बुनियादी ढांचे का निर्माण कर रहा था। कथन 2, योजना आयोग ने चैथी पंचवर्षीय योजना के बारे में कहा है कि यह लोगों, विशेष रूप से समाज के कम विशेषाधिकार प्राप्त वर्गों के जीवन स्तर को बढ़ाने के लिए एक सचेत, आंतरिक रूप से सुसंगत एवं सावधानी से योजनाबद्ध कार्यक्रम था। अधिभावी अंतःप्रेरणा सामाजिक न्याय की ज्वलंत भावना होनी चाहिए। वहीं बढ़ा हुआ उत्पादन भी अत्यधिक महत्व का है, तथा धन एवं आर्थिक शक्ति की एकाग्रता को हटाने या कम करने के लिए भी यह उतना ही महत्वपूर्ण है। (http://planningcommission.nic.in/plans/planrel/fiveyr/4th/4ppre.htm) कथन 3 के संबंध में, कोई स्पष्ट उल्लेख नहीं मिलता है।


download scanned papers here - Paper 1 Paper II


21. भारत के संविधान के संदर्भ में, निम्नलिखित कथनों पर विचार कीजिए :

  1. किसी भी केंद्रीय विधि को संवैधानिक रूप से अवैध घोषित करने की किसी भी उच्च न्यायालय की अधिकारिता नहीं होगी।
  2. भारत के संविधान के किसी भी संशोधन पर भारत के उच्चतम न्यायालय द्वारा प्रश्न नहीं उठाया जा सकता।

उपर्युक्त में से कौन-सा/से कथन सही है/हैं?

  1. केवल 1
  2. केवल 2
  3. 1 और 2 दोनों
  4. न तो 1, न ही 2

उत्तर (डी) कथन 2 स्पष्ट रूप से गलत है, सर्वोच्च न्यायालय संविधान में किसी संशोधन पर प्रश्न उठा सकता है यदि वह मूल संरचना सिद्धांत का उल्लंघन करता है। सबसे अच्छा उदाहरण 99 वां संशोधन, 2014 (एनजेएसी निर्णय) का है। इसलिए, विकल्प (बी) एवं (सी) सही नही हैं। अब, कथन 1 भी सही नही है क्योंकि भारत के किसी भी उच्च न्यायालय में इस तरह का कोई प्रतिबंध नहीं है। कानून की वर्तमान स्थिति, जो कि कुसुम इंगोट्स एंड अलसीस लिमिटेड बनाम भारत के मामले में विभिन्न उच्च न्यायालयों के निर्णयों द्वारा स्पष्ट हैं एवं जिसे सर्वोच्च न्यायालय द्वारा स्वीकार किया जाता है, यह है कि, केंद्रीय विधान की संवैधानिक वैधता उच्च न्यायालय द्वारा पारित अंतरिम या अंतिम आदेश पूरे देश में लागू होता है एवं इसलिए अन्य उच्च न्यायालयों को इस मुद्दे पर विपरीत विचार करने से रोकता है। यद्यपि यह सिद्धांत संविधान के अनुच्छेद 226 से मेल नही खाता, जोकि सभी व्यावहारिक उद्देश्यों के लिए कानूनन सही है।


22. निम्नलिखित कथनों पर विचार कीजिए :

  1. क्रय शक्ति समता {परचेज़िंग पॉवर पैरिटि (PPP)} विनिमय दरों की गणना विभिन्न देशों में एकसमान वस्तुओं और सेवाओं के मूल्यों की तुलना कर की जाती है।
  2. PPP डॉलर के संदर्भ में, भारत विश्व की छठी सबसे बड़ी अर्थव्यवस्था है।

उपर्युक्त में से कौन-सा/से कथन सही है/हैं?

  1. केवल 1
  2. केवल 2
  3. 1 और 2 दोनों
  4. न तो 1, न ही 2

उत्तर (ए) परिभाषानुसार, कथन 1 सही है। किंतु कथन 2 गलत है, क्योंकि भारतीय अर्थव्यवस्था अब पीपीपी डॉलर के आधार पर दुनिया में तीसरे स्थान पर है। इसलिए, उत्तर (ए) है। संदर्भ छवि यहां है।


23. भारत में पिछले पाँच वर्षों में खरीफ़ की फसलों की खेती के संदर्भ में, निम्नलिखित कथनों पर विचार कीजिए

  1. धान की खेती के अंतर्गत क्षेत्र अधिकतम हैं।
  2. ज्वार की खेती के अंतर्गत क्षेत्र, तिलहनों की खेती के अंतर्गत क्षेत्र की तुलना में अधिक हैं।
  3. कपास की खेती का क्षेत्र, गन्ने की खेती के क्षेत्र की तुलना में अधिक है।
  4. गन्ने की खेती के अंतर्गत क्षेत्र निरंतर घटा है।

उपर्युक्त में से कौन-से कथन सही हैं?

  1. केवल 1 और 3
  2. केवल 2, 3 और 4
  3. केवल 2 और 4
  4. 1, 2, 3 और 4

उत्तर (ए) कथन 2 सही नही है, क्योंकि ज्वार (मोटे अनाज में से एक) का बुवाई क्षेत्र निश्चित रूप से तिलहन के लिए अधिक नहीं है। मोटे अनाज कई छोटी अवधि के गर्म मौसम (खरीफ) की फसलों के व्यापक उप-समूह हैं जैसे कि ज्वार (सोरघम), बाजरा (पर्ल बाजरा), मक्का, रागी (फिंगर बाजरा) आदि। यदि 2 सही नही हैं, तो स्वतः ही उत्तर (ए) है। 1 एवं 3 दोनों सही हैं। संदर्भ छवि यहां है।

यहां पूर्ण नवीनतम डेटा - http://agricoop.gov.in/sites/default/files/2ndADVEST201819_E.pdf)


24. भारत द्वारा आयातित कृषि जिंसो में, पिछले पाँच वर्षों में निम्नलिखित में से किस एक का मूल्य के आधार पर अधिकतम आयात रहा है?

  1. मसाले
  2. ताजे फल
  3. दलहन
  4. वनस्पति तेल

उत्तर (डी) पिछले दशक में भारतीय खाद्य तेल आयात 8.4 फीसदी सीएजीआर से बढ़ा है। मोटे तौर पर, आयात में, परिष्कृत तेल का हिस्सा कुल आयात का 15 प्रतिशत रहा है, शेष कच्चा (गैर-परिष्कृत) खाद्य तेल आयात किया जाता है, जो स्थानीय इकाइयों में परिष्कृत होता है। यदि भारतीय उपज मौजूदा परिस्थिती से वैश्विक औसत तक भी बढ़ती है, तो भारतीय खाद्य तेल उत्पादन, बुआई के समान क्षेत्र पर ही, 67 फीसदी बढ़कर 13.5 मिलियन टन हो जाएगा। यदि पैदावार वैश्विक सर्वोत्तम दरों तक बढ़ती है, तो भारतीय खाद्य तेल उत्पादन मौजूदा 8-9 मिलीयन टन का 3 गुना हो जाएगा एवं भारत बुआई के समान क्षेत्र पर ही पर आत्मनिर्भर हो जाएगा! दालों के बारे में - दो साल के सूखे ने 2015 में दाल की कीमतों को बढ़ा दिया एवं नई दिल्ली को शुल्क-मुक्त आयात की अनुमति देने के लिए मजबूर किया। लेकिन 2016-17 के वित्तीय वर्ष में 6.6 मिलियन टन के रिकॉर्ड आयात से स्थानीय कीमतों में गिरावट आई। आयात पर अंकुश लगाने के अपने प्रयासों के तहत, सरकार ने 2018 में आयात कोटा तय करना शुरू कर दिया। भारत दुनिया का सबसे बड़ा उत्पादक एवं दालों का उपभोक्ता है। संदर्भ छवि यहां है।


25. राज्य-व्यवस्था के संदर्भ में, निम्नलिखित में से किस एक को आप स्वतंत्रता की सर्वाधिक उपयुक्त् व्याख्या के रूप में स्वीकार करेंगे?

  1. राजनीतिक शासकों की तानाशाही के विरूद्ध संरक्षण
  2. नियंत्रण का अभाव
  3. इच्छानुसार कुछ भी करने का अवसर
  4. स्वयं को पूर्णतः विकसित करने का अवसर

उत्तर (बी) राजनीति विज्ञान में, "लिबर्टी" शब्द लैटिन शब्द 'लिबर' से लिया गया है जिसका अर्थ है 'मुक्त'। मोटे तौर पर, स्वतंत्रता वह क्षमता है जिसमें व्यक्ति अपना मनचाहा कार्य कर सके। लेकिन इसका अर्थ यह नहीं लिया जा सकता है कि "जो कुछ भी पसंद है उसे करना" क्योंकि इससे अराजकता उत्पन्न हो सकती है। अतः विकल्प (c) गलत है। स्वतंत्रता निश्चित रूप से उन बंधनों से मुक्ति है जो व्यक्तियों को उनकी इच्छाओं के विरुद्ध बांधते हैं। अतः विकल्प (b) सबसे सही है। विकल्प (d) गलत नहीं है, लेकिन (c) बेहतर है। विकल्प (a) को सीधे खारिज किया जा सकता है क्योंकि यह स्वतंत्रता की एक निहित उपयोगिता है एवं इसकी परिभाषा नहीं ।


26. भारतीय रूपए की गिरावट रोकने के लिए निम्नलिखित में से कौन-सा एक सरकार/भारतीय रिज़र्व बैंक द्वारा किया जाने सर्वाधिक संभावित उपाय नहीं है?

  1. गैर-ज़रूरी वस्तुओं के आयात पर नियंत्रण और निर्यात को प्रोत्साहन
  2. भारतीय उधारकर्ताओं को रूपए मूल्यवर्ग के मसाला बॉन्ड जारी करने हेतु प्रोत्साहित करना
  3. विदेशी वाणिज्यिक उधारी से संबंधित दशाओं को आसान बनाना।
  4. एक प्रसरणशील मौद्रिक नीति का अनुसरण करना।

उत्तर (डी) यदि भारतीय रुपया फिसल रहा है, तो विकल्प (ए) उठाए जाने वाले पहले कदमों में से एक है। यह कीमती विदेशी मुद्रा के अनावश्यक खर्च को रोकता है। विकल्प (बी) एक अच्छा विचार है एवं साथ ही इससे डॉलर के उधार से बचा जा सकता है। विकल्प (सी) भी एक पसंदीदा मार्ग है, एवं इसमें अधिक डॉलर मिलता है (यह 2018 में किया गया था)। विकल्प (डी) सीधे समस्या से जुड़ा नहीं है। विस्तारवादी मौद्रिक नीति का मतलब ब्याज दरों में कमी (नरमी) है। इसका मतलब है कि स्थानीय प्रणाली में अधिक संभावित क्रेडिट अपटेक एवं क्रेडिट फ्लो, जिसके कारण रुपये का मूल्य आगे बढ़ सकता है (या ऐसा नहीं हो सकता है)।


27. निम्नलिखित कथनों पर विचार कीजिए :

’भुगतान प्रणाली आँकड़ों के भंडारण (स्टोरेज ऑफ पेमेंट सिस्टम डेटा)’ के संबंध में भारतीय रिजर्व बैंक के हाल का निदेश, जिसे प्रचलित रूप से डेटा डिक्टैट के रूप में जाना जाता है, भुगतान प्रणाली प्रदाताओं (पेमेंट सिस्टम प्रोवाइडर्स) को समादेशित करता है कि

  1. वे यह सुनिश्चित करेंगे कि उनके द्वारा संचालित भुगतान प्रणालियों से संबंधित समग्र आँकड़े एक प्रणाली के अंतर्गत केवल भारत में भंडारित किए जाएँ
  2. वे यह सुनिश्चित करेंगे कि इन प्रणालियों का स्वामित्व और संचालन सार्वजनिक क्षेत्र के उद्यम ही करें
  3. वे कैलेंडर वर्ष की समाप्ति तक भारत के नियंत्रक एवं महालेखापरीक्षक को समेकित प्रणाली लेखापरीक्षा रिपोर्ट प्रस्तुत करेंगे

उपर्युक्त में से कौन-सा/से कथन सही है/हैं?

  1. केवल 1
  2. केवल 1 और 2
  3. केवल 3
  4. 1, 2 और 3

उत्तर (ए) कथन 2 स्पष्ट रूप से गलत है, क्योंकि ऐसा कोई आदेश जारी नहीं किया गया था। अतः, विकल्प (बी) एवं (डी) सही नही हैं। कथन 3 भी सही नही है। इसलिए उत्तर (ए) - केवल 1 है। भुगतान प्रणाली प्रदाता यह सुनिश्चित करेंगे कि उनके द्वारा संचालित भुगतान प्रणालियों से संबंधित संपूर्ण डेटा केवल भारत में ही संग्रहीत हो। यही वह बात है जो डोनाल्ड ट्रम्प प्रशासन को भारत के बारे में बहुत परेशान करती है।


28. निम्नलिखित में से किसने अपने नागरिकों के लिए दत्त संरक्षण (डेटा प्रोटेक्शन) और प्रइवेसी के लिए ’सामान्य दत्त संरक्षण विनियमन (जेनरल डेटा प्रोटेक्शन रेगुलेशन)’ नामक एक कानून अप्रैल 2016 में अपनाया और उसका 25 मई, 2018 से कार्यान्वयन शुरू किया ?

  1. ऑस्ट्रेलिया
  2. कनाडा
  3. यूरोपीय संघ (यूरोपियन यूनियन)
  4. संयुक्त राज्य अमेरिका

उत्तर (सी) शायद सबसे आसान प्रश्नों में से एक! यूरोपीय संघ, 2018 में जीडीपीआर लाया एवं डिजिटल उद्यमों एवं डेटा प्रबंधन को नियंत्रित करने वाले नियमों को पूरी तरह से बदल दिया। संदर्भ छवि यहां है।


29.हाल ही में भारत ने निम्नलिखित में से किस देश के साथ ’नाभिकीय क्षेत्र में सहयोग क्षेत्रों के प्राथमिकीकरण और कार्यान्वयन हेतु कार्य योजना’ नामक सौदे पर हस्ताक्षर किया है?

  1. जापान
  2. रूस
  3. यूनाइटेड किंगडम
  4. संयुक्त राज्य अमेरिका

उत्तर (बी) रूसी संघ के राष्ट्रपति एच. ई. व्लादिमीर पुतिन की भारत यात्रा के दौरान, 5 अक्टूबर, 2018 को, भारत एवं रूस द्वारा संयुक्त रूप से परमाणु सहयोग क्षेत्रों के संचालन एवं कार्यान्वयन के लिए कार्य योजना पर, नई दिल्ली में हस्ताक्षर किए गए थे। भारत एवं रूस के बीच असैन्य परमाणु सहयोग की रणनीतिक साझेदारी, भारत की ऊर्जा-सुरक्षा एवं जलवायु परिवर्तन के पेरिस समझौते के तहत प्रतिबद्धताओं के लिए एक महत्वपूर्ण घटक है।


30. किसी अर्थव्यवस्था में मुद्रा गुणक (मनी मल्टिप्लायर) निम्नलिखित में से किस एक के साथ-साथ बढ़ता है?

  1. आरक्षित नकदी (कैश रिज़र्व) अनुपात में वृद्धि
  2. जनता की बैंकिंग आदतों में वृद्धि
  3. सांविधिक नकदी अनुपात में वृद्धि
  4. देश की जनसंख्या में वृद्धि

उत्तर (बी) यदि आप परिभाषानुसार जाते हैं, तो एक बहु-स्तरीय बैंकिंग प्रणाली के तहत मनी मल्टीप्लायर केंद्रीय बैंक के पैसे के वाणिज्यिक बैंक के पैसे का अनुपात है। केंद्रीय बैंक के पैसे का वास्तविक अनुपात, जिसे धन गुणक भी कहा जाता है, कम है, क्योंकि कुछ धन गैर-बैंक जनता द्वारा मुद्रा आदि के रूप में रखे जाते हैं, इसलिए, यदि जनसंख्या की बैंकिंग संबंधी आदते बदल जाती है, एवं वे मुख्यधारा बैंकिंग में प्रवेश करते हैं (अधिक खातों, आदि), तो गुणक में वृद्धि होगी। इसलिए, (बी) सही है। विकल्प (ए) एवं (सी) के विपरीत प्रभाव होंगे। विकल्प (डी) भी कुछ सार्थक नहीं दर्शाता है।



download scanned papers here - Paper 1 Paper II


31. भारत में विशिष्टः असुरक्षित जनजातीय समूहों {पर्टिकुलरली वल्नरेबल ट्राइबल ग्रुप्स (PVTGs)} के बारे में, निम्नलिखित कथनों पर विचार कीजिए :

  1. PVTGs देश के 18 राज्यों तथा एक संघ राज्यक्षेत्र में निवास करते हैं।
  2. स्थिर या कम होती जनसंख्या, PVTG स्थिति के निर्धारण के मानदंडो में से एक है।
  3. देश में अब तक 95 PVTGs आधिकारिक रूप से अधिसूचित हैं।
  4. PVTGs की सूची में ईरूलार और कोंडा रेड्डी जनजातियाँ शामिल की गइ हैं।

उपर्युक्त में से कौन-से कथन सही है?

  1. 1, 2 और 3
  2. 2, 3 और 4
  3. 1, 2 और 4
  4. 1, 3 और 4

उत्तर (सी) कथन 1 सही है क्योंकि पीवीटीजीस 18 राज्यों एवं अंडमान निकोबार द्वीप समूह के केंद्र-शासित प्रदेश में रहते हैं। इसका मतलब है कि उत्तर विकल्प (बी) नहीं हो सकता है। कथन 3 सही नही है क्योंकि देश में अब तक आधिकारिक तौर पर अधिसूचित 75 पीवीटीजीस हैं। अतः (ए) एवं (डी) को खारिज किया जा सकता है। इसलिए, हमारा उत्तर (सी) है। संदर्भ छवि यहां है।


32. भारत के संविधान और संदर्भ में, सामान्य विधियों में अंतर्विष्ट प्रतिषेध अथवा निर्बंधन अथवा उपबंध, अनुच्छेद 142 के अधीन सांविधानिक शक्तियों पर प्रतिषेध अथवा निर्बंधन की तरह कार्य नहीं कर सकते। निम्नलिखित में से कौन-सा एक, इसका अर्थ हो सकता है?

  1. भारत के निर्वाचन आयोग द्वारा अपने कर्तव्यों का निर्वहन करते समय लिए गए निर्णयों को किसी भी न्यायालय में चुनौती नहीं दी जा सकती।
  2. भारत का उच्चतम न्यायालय अपनी शक्तियों के प्रयोग में संसद द्वारा निर्मित विधियों से बाध्य नहीं होता।
  3. देश में गंभीर वित्तीय संकट की स्थिति में, भारत का राष्ट्रपति मंत्रिमंडल के परामर्श के बिना वित्तीय आपात घोषित कर सकता है
  4. कुछ मामलों में राज्य विधानमंडल, संघ विधानमंडल की सहमति के बिना, विधि निर्मित नहीं कर सकते।

उत्तर (बी) भारत के संविधान का अनुच्छेद 142 अपने फरमानों एवं आदेशों को लागू करने के लिए सर्वोच्च न्यायालय को व्यापक अधिकार देता है। विकल्प (ए) अप्रासंगिक है। विकल्प (सी) अप्रासंगिक है क्योंकि यह अनुच्छेद 360 के तहत कवर किया गया है। विकल्प (डी) भी अप्रासंगिक है, एवं इसे अनुच्छेद 246 के तहत कवर किया गया है। यदि आपको केवल अनुच्छेद तथा वे किससे संबंधित हैं, यह याद हैं, तो आप तीन विकल्पों को सीधे खारिज कर सकते थे। संदर्भ छवि यहां है।


33. भारत के किसी राज्य की विधान सभा के संदर्भ में, निम्नलिखित कथनों पर विचार कीजिए :

  1. वर्ष के प्रथम सत्र के प्रारंभ में राज्यपाल सदन के सदस्यों के लिए रूढिवगत संबोधन करता है।
  2. जब किसी विशिष्ट विषय पर राज्य विधानमंडल के पास कोई नियम नहीं होता, तो उस विषय पर वह लोग सभा के नियम का पालन करता है।

उपर्युक्त में से कौन-सा/से कथन सही है/हैं?

  1. केवल 1
  2. केवल 2
  3. 1 और 2 दोनों
  4. न तो 1, न ही 2

उत्तर (सी) भारतीय संविधान के अनुच्छेद 176 (1) में कहा गया है कि राज्यपाल विधानसभा के प्रत्येक आम चुनाव के बाद एवं प्रत्येक वर्ष के पहले सत्र के शुरू होने के बाद पहले सत्र की शुरुआत में दोनों सदनों को एक साथ संबोधित करेंगे तथा विधायिका को उसके इस प्रकार बुलाए जाने का कारण बताएंगे। इसलिए (1) सत्य है। इसलिए विकल्प (बी) एवं (डी) सही नही हैं। अब कथन 2 स्थापित परिपाटी के संदर्भ में सही है। इसलिए, उत्तर (सी) है।


34. निम्नलिखित कथनों पर विचार कीजिए :

  1. भ्रष्टाचार के विरूद्ध संयुक्त राष्ट्र कन्वेंशन {यूनाइटेड नेशंस कन्वेंशन अगेंस्ट करप्शन (UNCAC)} का ’भूमि, समुद्र और वायुमार्ग से प्रवासियों की तस्करी के विरूद्ध एक प्रोटोकॉल’ होता है।
  2. UNCAC अब तक का सबसे पहला विधितः बाध्यकारी सार्वभौम भ्रष्टाचार-निरोधी लिखत है।
  3. राष्ट्र-पार संगठित अपराध के विरूद्ध संयुक्त राष्ट्र कन्वेंशन {यूनाइटेड नेशंस कन्वेंशन अगेंस्ट ट्रांसनैशनल ऑर्गेनाइज़्ड क्रइम UNTOC)} की एक विशिष्टता ऐसे एक विशिष्ट अध्याय का समावेशन है, जिसका लक्ष्य उन संपत्तियों को उनके वैध स्वामियों को लौटाना है जिनसे वे अवैध तरीके से ले ली गई थीं।
  4. मादक द्रव्य और अपराध विषयक संयुक्त राष्ट्र कार्यालय {यूनाइटेड नेशंस ऑफिस ऑन ड्रग्स ऐंड क्राइम (UNODC)} संयुक्त राष्ट्र के सदस्य राज्यों द्वारा UNCAC और UNTOC दोनो के कार्यान्वयन में सहयोग करने के लिए अधिदेशित हैं।

उपर्युक्त में से कौन-से कथन सही है/हैं?

  1. केवल 1 और 3
  2. केवल 2, 3 और 4
  3. केवल 2 और 4
  4. 1, 2, 3 और 4

उत्तर (सी) वक्तव्य 1 सही नही है क्योंकि संयुक्त राष्ट्र कन्वेंशन फॉर ट्रांसनेशनल ऑर्गनाइज्ड क्राइम (युएनटीओसी) में भूमि, समुद्र एवं वायु द्वारा प्रवासियों की घुसपैठ के खिलाफ प्रोटोकॉल है। अतः, विकल्प (ए) एवं (डी) सही नही हैं। अब कथन 3 की जांच करें। यह भ्रष्टाचार के खिलाफ संयुक्त राष्ट्र सम्मेलन (युएनसीएसी) के लिए सही है एवं युएनटीओसी के लिए नहीं, अतः यह सही नही है। अतः, विकल्प (बी) सही नही है। इसलिए, (सी) सही है। यह प्रश्न पेचीदा था, एवं वैश्विक आतंकवाद, एफएटीएफ से जुड़े घटनाक्रम, वैश्विक भ्रष्टाचार के मामले आदि के संदर्भ में पूछा गया था। संदर्भ छवि यहां है।


35. निम्नलिखित कथनों पर विचार कीजिए :

  1. भारतीय वन अधिनियम, 1927 में हाल में हुए संशोधन के अनुसार, वन निवासियों को वन क्षेत्रों में उगने वाले बाँस को काट गिराने का अधिकार है।
  2. अनुसूचित जनजाति एवं अन्य पारंपरिक वनवासी (वन अधिकारों की मान्यता) अधिनियम, 2006 के अनुसार, बाँस एक गौण वनोपज है।
  3. अनुसूचित जनजाति एवं अन्य पारंपरिक वनवासी (वन अधिकारों की मान्यता) अधिनियम, 2006, वन निवासियों को गौण वनोपज के स्वामित्व की अनुमति देता है।

उपर्युक्त में से कौन-सा/से कथन सही है/हैं?

  1. केवल 1 और 2
  2. केवल 2 और 3
  3. केवल 3
  4. 1, 2 और 3

उत्तर (ए) तीनों कथन सही हैं। भारतीय वन अधिनियम, 1927 की धारा 2 (7) में संशोधन के बाद, बांस अब एक पेड़ नहीं है एवं गिर गया बांस लकड़ी नहीं है (पहले बांस को लकड़ी माना जाता था)। तो लाखों किसानों द्वारा निजी या गृहस्थ भूमि में उगाए गए किसी भी बांस को किसी भी राज्य वन विभाग से फेलिंग अनुमति या पारगमन की आवश्यकता नहीं होती है। अतः कथन 1 सही है। कथन 2 - अनुसूचित जनजातियों एवं अन्य पारंपरिक वन निवासियों (वन अधिकारों की मान्यता) अधिनियम, 2006 के तहत ‘लघु वनोपज’ में पौध-उत्पत्ति कीसभी गैर-लकड़ी वन उपज शामिल है जिनमें बांस, ब्रश की लकड़ी, स्टंप, बेंत, ज्वार, कोकून, शहद, मोम, लाख, तेंदू या केंदू के पत्तों, औषधीय पौधों एवं जड़ी बूटियों, जड़, कंद इत्यादि शामिल है। अतः कथन 2 भी सही है। कथन 3 भी सही है। संदर्भ छवि यहां है।


36.भारत के संविधान का कौन-सा अनुच्छेद अपनी पसंद के व्यक्ति से विवाह करने के किसी व्यक्ति के अधिकार को संरक्षण देता है?

  1. अनुच्छेद 19
  2. अनुच्छेद 21
  3. अनुच्छेद 25
  4. अनुच्छेद 29

उत्तर (बी) ऑनर किलिंग के संदर्भ में, शक्ति वाहिनी, एक एनजीओ, ने भारतीय सुप्रीम कोर्ट का दरवाजा खटखटाया था, तथा राज्य सरकारों एवं केंद्र सरकार से इनसे निपटने के लिए निवारक कदम उठाने के लिए, निर्देश मांगे थे। भारत के सर्वोच्च न्यायालय की एक पीठ ने 2018 में एक फैसले में कहा कि - ‘आनर किलिंग, व्यक्तिगत स्वतंत्रता, अभिव्यक्ति की स्वतंत्रता एवं तथा चुनने की स्वतंत्रता की स्वयं-धारणा, की हत्या है। यह ध्यान में रखना होगा कि जब दो वयस्क एक-दूसरे को जीवन साथी के रूप में चुनते हैं, तो यह उनकी पसंद की अभिव्यक्ति है जिसे संविधान के अनुच्छेद 19 एवं 21 के तहत मान्यता प्राप्त है। इस तरह के अधिकार को संवैधानिक कानून की मंजूरी है एवं एक बार मान्यता प्राप्त होने के बाद, उक्त अधिकार को संरक्षित करने की आवश्यकता है ... । इस प्रश्न में मुख्य शब्द ‘सुरक्षा’ है। जबकि अनुच्छेद 19 नागरिक को यह स्वतंत्रता देता है, अनुच्छेद 21 उस स्वतंत्रता की रक्षा करने के लिए है। जीवन एवं स्वतंत्रता का अधिकार कहता है कि ‘कोई भी व्यक्ति अपने जीवन या व्यक्तिगत स्वतंत्रता से कानून के दायरों में रहते हुए, वंचित नहीं रहेगा।


37. निम्नलिखित कथनों पर विचार कीजिए :

  1. भारतीय पेटेंट अधिनियम के अनुसार, किसी बीज को बनाने की जैव प्रक्रिया को भारत में पेटेंट कराया जा सकता है।
  2. भारत में कोई बौद्धिक संपदा अपील बोर्ड नहीं है।
  3. पादप किस्में भारत में पेटेंट कराए जाने के पात्र नहीं है।

उपर्युक्त में से कौन-सा/से कथन सही है/हैं?

  1. केवल 1 और 3
  2. केवल 2 और 3
  3. केवल 3
  4. 1, 2 और 3

उत्तर (सी) कथन 2 सही नही है, क्योंकि आईपीएबी मौजूद है, एवं कुछ बड़े मामलों में खबरों में रहा है। भारतीय बौद्धिक संपदा अपीलीय बोर्ड (आईपीएबी) का गठन 15 सितंबर, 2003 को भारतीय ट्रेडमार्क अधिनियम, 1999 एवं भौगोलिक संकेतक (माल/पंजीकरण एवं संरक्षण) अधिनियम, 1999 के तहत रजिस्ट्रार के निर्णयों के खिलाफ अपीलों को सुनने एवं हल करने के लिए भारत सरकार द्वारा किया गया था। इसलिए, विकल्प (बी) एवं (डी) को खारिज किया जा सकता है। कथन 1 सही नहीं है। पेटेंट अधिनियम, 1970 के पृष्ठ 9 पर बिंदु (जे) का संदर्भ लें। कथन 3 सही है। भारत द्वारा विश्व व्यापार संगठन के तहत ट्रिप्स पर हस्ताक्षर करने के बाद, इसने संयंत्र प्रजनकों एवं किसानों के अधिकार अधिनियम, 2001 (पीपीवी - एफआर अधिनियम 2001) को अधिनियमित किया, संयंत्र प्रजनकों, शोधकर्ताओं एवं किसानों को आईपी संरक्षण प्रदान करने के लिए जिन्होंने कोई नई या विलुप्त होने वाली किस्मों को विकसित किया है। भारत प्लांट वेराइटी के लिए पेटेंट नहीं देता है, लेकिन इसमें सुई जेनिस (अद्वितीय) प्रणाली है जो भारतीय किसानों एवं प्रजनकों के लिए अधिक सुविधाजनक है। इसलिए, हमारा उत्तर (सी) है । संदर्भ छवि यहां है।


38. निम्नलिखित कथनों पर विचार कीजिए :

पर्यावरण संरक्षण अधिनियम, 1986 भारत सरकार को सशक्त करता है कि

  1. वह पर्यावरणीय संरक्षण की प्रक्रिया में लोक सहभागिता की आवश्यकता का और इसे हासिल करने की प्रक्रिया और रीति का विवरण दें
  2. वह विभिन्न स्त्रोतों से पर्यावरणीय प्रदूषकों के उत्सर्जन या विसर्जन के मानक निर्धारित करें।

उपर्युक्त में से कौन-सा/से कथन सही है/हैं?

  1. केवल 1
  2. केवल 2
  3. 1 और 2 दोनों
  4. न तो 1, न ही 2

उत्तर (बी) कथन 1 सही नहीं है, क्योंकि इसमें इपीए 1986 उल्लेख कहीं भी नहीं है। कथन 2 वास्तव में सही है। पर्यावरण संरक्षण अधिनियम, 1986 के अध्याय 2 का संदर्भ लें। संदर्भ छवि यहां है।


39. भारत में ठोस अपशिष्ट प्रबंधन नियम, 2016 के अनुसार, निम्नलिखित में से कौन-सा एक कथन सही हैं?

  1. अपशिष्ट उत्पादक को पाँच कोटियों में अपशिष्ट अलग-अलग करने होंगे।
  2. ये नियम केवल अधिसूचित नगरीय स्थानीय निकायों, अधिसूचित नगरों तथा सभी औद्योगिक नगरों पर ही लागू होंगें
  3. इन नियमों में अपशिष्ट भराव स्थलों तथा अपशिष्ट प्रसंस्करण सुविधाओं के लिए सटीक और ब्यौरेवार मानदंड उपबंधित हैं।
  4. अपशिष्ट उत्पादक के लिए यह आज्ञापक होगा कि किसी एक जिले में उत्पादित अपशिष्ट, किसी अन्य जिले में न ले जाया जाए।

उत्तर (सी) सभी ‘अपशिष्ट निर्माता’ अपने द्वारा उत्पन्न कचरे को तीन अलग तरह से जमा करेंगे, जैसे कि बायो डिग्रेडेबल, नॉन-बायो डिग्रेडेबल एवं घरेलू खतरनाक कचरा तथा उन्हें उपयुक्त डिब्बे या थैली में बंद करके अधिकृत अपशिष्ट संकलनकर्ता को देंगे। विकल्प (ए) सही नहीं है। नियम प्रत्येक शहरी-स्थानीय निकाय, शहर के बाहरी इलाके, भारत के रजिस्ट्रार जनरल एवं जनगणना आयुक्त द्वारा अधिसूचित कस्बे, अधिसूचित औद्योगिक टाउनशिप, भारतीय रेलवे के नियंत्रण वाले क्षेत्र, हवाई अड्डे, एयरबेस, पोर्ट एवं बंदरगाह, रक्षा प्रतिष्ठान, विशेष आर्थिक क्षेत्र, राज्य एवं केंद्र सरकार के संगठन, तीर्थयात्रियों के स्थान, धार्मिक एवं ऐतिहासिक महत्व , जो संबंधित राज्य सरकार द्वारा समय-समय पर घोषित किए जाते है एवं औद्योगिक अपशिष्ट, खतरनाक अपशिष्ट, खतरनाक रसायन, जैव चिकित्सा अपशिष्ट, ई, कचरा, सीसा एसिड बैटरी एवं रेडियो सक्रिय अपशिष्ट को छोड़कर प्रत्येक घरेलू, संस्थागत, वाणिज्यिक एवं किसी भी तरह के अन्य गैर आवासीय ठोस अपशिष्ट निर्माताओं के लिए लागु है। विकल्प (बी) सही नहीं है। कानून में अंतर-जिला संचार का उल्लेख नहीं है। फाइल देखें। विकल्प (सी) सही है। संदर्भ छवि यहां है।


40. निम्नलिखित कथनों पर विचार कीजिए :

औद्योगिक रोजगार (स्थायी आदेश) केंद्रीय (संशोधन) नियम, 2018 के अनुसार

  1. यदि नियत अवधि रोजगार के लिए नियमों को कार्यान्वित किया जाता है, तो फर्म/कंपनियों के लिए कामगारों की छँटनी करना अपेक्षाकृत आसान हो जाता है।
  2. अस्थायी कामगारों के मामले में रोजगार समाप्त करने के लिए कोई नोटिस देना आवश्यक नहीं होगा।

उपर्युक्त में से कौन-सा/से कथन सही है/हैं?

  1. केवल 1
  2. केवल 2
  3. 1 और 2 दोनों
  4. न तो 1, न ही 2

उत्तर (सी) यह स्पष्ट है कि काम पर रखने एवं निकालने में अधिक लचीलापन लाने के लिए, मानदंडों को बदला गया है। कथन 1 सही है। मॉडल स्थायी आदेश के उप-पैरा (बी) के लिए अनुच्छेद 13 में, निम्न प्रकार लिखीत है - ‘निश्चित अवधि के रोजगार पर कार्य करने वाले किसी कर्मचारी को अनुबंध के नवीनीकरण ना किए जाने या इस प्रकार के अनुबंध के समाप्त हो जाने पर उसे पुनः नवीनीकृत ना किए जाने के मामले में नियोक्ता किसी भी पूर्वसूचना देनेया उसके एवज में भुगतान करने के लिए जिम्मेदार नही होगा। अतः, कथन 2 भी सही है।


41. मनोरंजन हेतु डिजिटल प्रौद्योगिकियों के संदर्भ में, निम्नलिखित कथनों पर विचार कीजिए :

  1. संवर्धित वास्तविकता {ऑगमेंटेड रिएलिटि (AR)} में एक छद्म वातावरण सृजित हो जाता है और भौतिक संसार पूरी तरह बहिष्कृत हो जाता है।
  2. आभासी वास्तविकता {वर्चुअल रिएलिटि (VR)} में कम्प्यूटर द्वारा सृजित प्रतिमाएँ वास्तविक जीवन की वस्तुओं या परिवेशों पर प्रक्षेपित हो जाती है।
  3. AR व्यक्तियों को संसार में विद्यमान रहने देता है और स्मार्ट-फोन या PC के कैमरे का उपयोग कर अनुभव को उन्नत करता है।
  4. VR संसार को पृथक् कर देता है और व्यक्ति को एक अलग धरातल पर ले जाकर उसे पूर्ण निमग्नता का अनुभव प्रदान करता है

उपर्युक्त में से कौन-सा/से कथन सही है/हैं?

  1. केवल 1 और 2
  2. केवल 3 और 4
  3. 1, 2 और 3
  4. केवल 4

उत्तर (बी) सबसे पहले, हमें याद रखना चाहिए कि वर्चुअल रियलिटी या वीआर या संवर्धित वास्तविकता वह वैकल्पिक संसार है जिसमें उपयोगकर्ता पूर्णतः खो जाता है। इसलिए, कथन 4 निश्चित रूप से सही है। इसलिए, विकल्प (ए) एवं (सी) सही नही हैं। संवर्धित वास्तविकता किसी वस्तु की छवि पर डिजिटल जानकारी जोड़ने तथा प्रौद्योगिकी के उपयोग द्वारा निर्मित ‘वास्तविकता’ का एक उन्नत संस्करण है। अब, कथन 3 की जाँच करें। यह सही भी है। इसलिए, सबसे अच्छा उत्तर विकल्प (बी) है।


42. शब्द ’डेनिसोवन (Denisovan)’ कभी-कभी समाचार माध्यमों में किस संदर्भ में आता है?

  1. एक प्रकार के डायनासोर का जीवाश्म
  2. एक आदिमानव जाति (स्पीशीज़)
  3. पूवोत्तर भारत में प्राप्त एक गुफा तंत्र
  4. भारतीय उपमहाद्वीप के इतिहास में एक भूवैज्ञानिक कल्प

उत्तर (बी) यह शब्द अब एक विलुप्त प्रजाति या जीनस होमो में पुरातन मनुष्यों की उप-प्रजातियों से संबंधित है। इसने पिछले एक साल में खबर बनाई क्योंकि तिब्बत में जीवाश्म पाए गए एवं वे भी 1.5 लाख साल से भी अधिक समय पहले के। यह प्रश्न नियमित अखबार पढ़ने से हल किया जा सकता था। संदर्भ छवि यहां है।


43. विज्ञान में हुए अभिनव विकासों के संदर्भ में, निम्नलिखित में से कौन-सा एक कथन सही नहीं है?

  1. विभिन्न जातियों की कोशिकाओं से लिए गए DNA के खंडो को जोड़कर प्रकार्यात्मक गुणसूत्र रचे जा सकते हैं।
  2. प्रयोगशालाओं में कृत्रिम प्रकार्यात्मक DNA के हिस्से रचे जा सकते हैं।
  3. किसी जंतु कोशिका से निकाले गए DNA के किसी हिस्से को जीवित कोशिका से बाहर, प्रयोगशाला में, प्रतिकृत कराया जा सकता है।
  4. पादपों और जंतुओं से निकाली गई कोशिकाओं में प्रयोगशाला की पेट्री डिश में कोशिका विभाजन कराया जा सकता है।

उत्तर (-) पुनर्संयोजक डीएनए (आरडीएनए) अणु आनुवंशिक पुनर्संयोजन (जैसे आणविक क्लोनिंग) की प्रयोगशाला विधियों द्वारा गठित डीएनए अणु हैं, जो कई स्रोतों से आनुवंशिक सामग्री को एक साथ लाते हैं, जिससे अनुक्रम बनते हैं जो अन्यथा जीनोम में नहीं मिलेंगे। पुनरावर्ती डीएनए अणुओं को कभी-कभी काइमेरिक डीएनए कहा जाता है, क्योंकि वे दो अलग-अलग प्रजातियों की सामग्री से बने हो सकते हैं, जैसे पौराणिक काईमेरा। तो, विकल्प (ए) सही है। मई 2019 में, कैम्ब्रिज विश्वविद्यालय के वैज्ञानिकों ने मानव निर्मित डीएनए - बैक्टीरिया ई कोलाई के साथ एक जीवित जीव पैदा करने का दावा किया है। पूरा जीन सिंथेटिक था। तो, (बी) भी सही है। विकल्प (सी) भी सही है क्योंकि यह नियमित रूप से किया जाता है। (संदर्भ - https://courses.lumenlearning.com/boundless-biology/chapter/dna-replication/) । विकल्प (डी) भी सही है। विकल्प (बी) के लिए संदर्भ छवि यहां है।


44. निम्नलिखित कथनों पर विचार कीजिए :

डिजिटल हस्ताक्षर

  1. एक ऐसा इलेक्ट्रानिक अभिलेख है, जो इसे जारी करने वाले प्रमाणन प्राधिकारी की पहचान करता है।
  2. इंटरनेट पर सूचना या सर्वर तक पहुँच के लिए किसी व्यक्ति की पहचान के प्रमाण के रूप में प्रयुक्त होता है।
  3. इलेक्ट्रॉनिक दस्तावेज़ पर हस्ताक्षर करने की एक इलेक्ट्रॉनिक पद्धति है और सुनिश्चित करता है कि मूल अंश अपरिवर्तित है

उपर्युक्त में से कौन-सा/से कथन सही है/हैं?

  1. केवल 1
  2. केवल 2 और 3
  3. केवल 3
  4. 1, 2 और 3

उत्तर (बी) डिजिटल हस्ताक्षर किसी हस्ताक्षर का एक इलेक्ट्रॉनिक रूप है जिसका उपयोग किसी संदेश के प्रेषक या दस्तावेज के हस्ताक्षरकर्ता की पहचान को प्रमाणित करने के लिए किया जा सकता है, एवं यह भी सुनिश्चित करता है कि संदेश या दस्तावेज की मूल सामग्री अपरिवर्तित है। डिजिटल हस्ताक्षर आसानी से परिवहनीय हैं एवं किसी अन्य द्वारा नकल नहीं किए जा सकते हैं। इसमें यह सुनिश्चित करने की क्षमता है कि मूल हस्ताक्षरित संदेश पहुंचा है तथा प्रेषक बाद में इसे आसानी से अस्वीकार नहीं कर सकता है। https://www.e-mudhra.com/faq.html) इसलिए, कथन 2 एवं 3 सही हैं। लेकिन, डिजिटल हस्ताक्षर ‘डिजिटल हस्ताक्षर प्रमाणपत्र’ (डीएससी) से अलग हैं। एक डीएससी भौतिक या कागजी प्रमाण पत्र का एक इलेक्ट्रॉनिक प्रारूप है जैसे ड्राइविंग लाइसेंस, पासपोर्ट आदि। एक डिजिटल हस्ताक्षर एक इलेक्ट्रॉनिक दस्तावेज पर हस्ताक्षर करने का एक इलेक्ट्रॉनिक तरीका है जबकि एक डिजिटल हस्ताक्षर प्रमाणपत्र एक कंप्यूटर आधारित रिकॉर्ड है जो इसे जारी करने वाले प्रमाणन प्राधिकरण की पहचान करता है। इसलिए, कथन 1 सही नही है। तो, सही उत्तर विकल्प (बी) है।


45. परिधेय प्रौद्योगिकी (विअरेबल टेक्नोलॉजि) के संदर्भ में, परिधेय उपकरणों द्वारा निम्नलिखित में से कौन-सा/से कार्य निष्पन्न किया जा सकता है/किए जा सकते हैं?

  1. किसी व्यक्ति का अवस्थान (लोकेशन) निर्धारण
  2. किसी व्यक्ति का निद्रा मॉनीटरन
  3. श्रवण दोषयुक्त व्यक्ति की सहायता

नीचे दिए गए कूट का प्रयोग कर सही उत्तर चुनिए।

  1. केवल 1
  2. केवल 2 और 3
  3. केवल 3
  4. 1, 2 और 3

उत्तर (डी) ‘पहनने योग्य तकनीक’ बताई गई सभी चीजों को कर सकती है। यह स्मार्ट इलेक्ट्रॉनिक उपकरणों (माइक्रो-कंट्रोलरों के साथ इलेक्ट्रॉनिक उपकरण) के रूप में फैशन तकनीक, टेक टॉग्स या फैशन इलेक्ट्रॉनिक्स है, जिसे कपड़े या शरीर पर पहना जा सकता है। पहनने योग्य उपकरण, जैसे - गतिविधि ट्रैकर, ‘इंटरनेट ऑफ थिंग्स’ के रूप में कार्य करते हैं। इसमें लोकेशन आइडेंटिफिकेशन (जीपीएस के जरिए), स्लीप मॉनिटरिंग, हियरिंग एड आदि सहित कई तरह के एप्लिकेशन हैं। इस फील्ड का विस्तार तेजी से हो रहा है।


46. ’RNA अंतर्क्षेप {RNA इंटरफेरेंस (RNAi)} प्रौद्योगिकी ने पिछले कुछ वर्षों में लोकप्रियता हासिल कर ली है। क्यों?

  1. यह जीन अनभिव्यक्तिकरण (जीन साइलेंसिंग) रोगोपचारों के विकास में प्रयुक्त होता है।
  2. इसे कैंसर की चिकित्सा में रोगोपचार विकसित करने हेतु प्रयुक्त किया जा सकता है।
  3. इसे हॉर्मोन प्रतिस्थापन रोगोपचार विकसित करने हेतु प्रयुक्त किया जा सकता है
  4. इसे ऐसी फसल पादपों को उगाने के लिए प्रयुक्त किया जा सकता है, जो विषाणु रोगजनकों के लिए प्रतिरोधी हो।

नीचे दिए गए कूट का प्रयोग कर सही उत्तर चुनिए।

  1. 1, 2 और 4
  2. 2 और 3
  3. 1 और 3
  4. केवल 1 और 4

उत्तर (ए) आरएनए हस्तक्षेप (आरएनएआई) एक जैविक प्रक्रिया है जिसमें आरएनए अणु लक्षित एमआरएनए अणुओं को बेअसर करते हैं, एवं जीन अभिव्यक्ति (जीन सिलिंग) को रोकते हैं। तकनीकी रूप से, आरएनए हस्तक्षेप (आरएनएआई) या पोस्ट-ट्रांसक्रिप्शनल जीन सिलिंग (पीटीजीएस) डबल-स्ट्रेन्ड आरएनए के लिए एक संरक्षित जैविक प्रतिक्रिया है जो अंतर्जात परजीवी एवं बहिर्जात रोगजनक न्यूक्लिक एसिड के लिए प्रतिरोध की मध्यस्थता करता है, एवं प्रोटीन-कोडिंग जीन की अभिव्यक्ति को नियंत्रित करता है। अनुक्रम-विशिष्ट जीन साइलेंसिंग के लिए यह प्राकृतिक तंत्र प्रयोगात्मक जीव विज्ञान में क्रांतिकारी बदलाव का वादा करता है एवं कार्यात्मक जीनोमिक्स, चिकित्सीय हस्तक्षेप, कृषि एवं अन्य क्षेत्रों में महत्वपूर्ण व्यावहारिक अनुप्रयोग है। 3 को छोड़कर, अन्य सभी कथन सही हैं।

आरएनए एवं डीएनए न्यूक्लिक एसिड हैं, एवं, लिपिड, प्रोटीन एवं कार्बोहाइड्रेट के साथ, जीवन के सभी ज्ञात रूपों के लिए आवश्यक चार प्रमुख माईक्रोमाॅलिक्युल शर्तों का गठन करते हैं। डीएनए की तरह, आरएनए को न्यूक्लियोटाइड्स की एक श्रृंखला के रूप में इकट्ठा किया जाता है, लेकिन डीएनए के विपरीत यह अधिक बार प्रकृति में पाया जाता है क्योंकि एक जोड़ी-स्ट्रैंड के बजाय स्वयं पर एकल-स्ट्रैंड पर मुड़ा हुआ है।


47. हाल ही में वैज्ञानिकों ने पृथ्वी से अरबों प्रकाश-वर्ष दूर विशालकाय ’ब्लैकहोलों’ के विलय का प्रेक्षण किया इस प्रेक्षण का क्या महत्व है?

  1. ’हिग्स बोसॉन कणों’ का अभिज्ञान हुआ ।
  2. ’गुरूत्वीय तरंगो’ का अभिज्ञान हुआ।
  3. ’वॉर्महोल’ से होते हुए अंतरा-मंदाकिनीय अंतरिक्ष यात्रा की संभावना की पुष्टि हुई।
  4. इसने वैज्ञानिकों को ’विलक्षणता (सिंगुलैरिटि)’ को समझना सुकर बनाया।

उत्तर (बी) दिसंबर 2018 में, वैज्ञानिकों ने पाया कि अरबों प्रकाश वर्ष दूर, दो ब्लैक होल के टकराने से एक बड़ा ब्लेक होल बना है - यह अब तक पता चला सबसे बड़ा ब्लैक होल है। इसका द्रव्यमान सूर्य की तुलना में 80 गुना अधिक था। स्पेसटाइम के फैब्रिक में इंजेक्ट की गई ऊर्जा भी रिकॉर्ड ब्रेकिंग थी, जिसमें पांच सूर्य के द्रव्यमान के बराबर ऊर्जा को गुरुत्वाकर्षण तरंगों के रूप में मुक्त की गई। अमेरिका में एडवांस्ड लेजर इंटरफेरोमीटर ग्रेविटेशनल-वेव ऑब्जर्वेटरी (एआईएलआईजीओ) एवं एडवांस्ड विर्गो फेसिलीटी, इटली द्वारा किए गए अनुसंधानों के संग्रह के जारी किए गए हिस्से में तीन अन्य छोटे पृथक ब्लैक होल टकरावों की घटना के साथ इस घटना का विवरण है। यह फेसिलीटी गुरुत्वाकर्षण तरंगों का पता लगाने में मदद करती है। आप विकल्प (सी) को सीधे अलग कर सकते हैं, क्योंकि यह शुद्ध विज्ञान-गल्प है। विकल्प (ए) संबंधित नहीं है। संदर्भ छवि यहां है।


48.निम्नलिखित में से कौन-से, भारत में सूक्ष्मजैविक रोजगनकों में बहु-औषध प्रतिरोध होने के कारण हैं?

  1. कुछ व्यक्तियों में आनुवांशिक पूर्ववृत्ति (जेनेटिक प्रीडिस्पोजीशन) का होना
  2. रोगों के उपचार के लिए प्रतिजैविकों (ऐंटिबॉयोटिक्स) की गलत खुराकें लेना
  3. पशुधन फार्मिंग में प्रतिजैविकों का इस्तेमाल करना
  4. कुछ व्यक्तियों में चिरकालिक रोगों की बहुलता होना

नीचे दिए गए कूट का प्रयोग कर सही उत्तर चुनिए।

  1. 1 और 2
  2. केवल 2 और 3
  3. 1, 3, और 4
  4. 2, 3 और 4

उत्तर (बी) भारत (एवं दुनिया भर में) एंटीबायोटिक दवाओं के बड़े पैमाने पर उपयोग (दुरुपयोग) ने एएमआर या एबीआर की खतरनाक स्थिति पैदा की है। मानव चिकित्सा के साथ ही पालतु जानवरों एवं यहां तक कि एक्वाकल्चर में मछली के लिए इस्तेमाल होने वाली एंटीबायोटिक से रोगजनक बैक्टीरिया, दवाओं की बड़ी मात्रा के लिए प्रतिरोधी हो गया है। रोगाणुरोधी प्रतिरोध (एआर या एबीआर), माइक्रोब की, उस दवा के प्रभाव का विरोध करने की क्षमता है, जो पहले उस माइक्रोब को हटाने में सफल थी। एंटीबायोटिक प्रतिरोध (एआर या एबीआर) शब्द एएमआर का सबसेट है, क्योंकि यह केवल एंटीबायोटिक दवाओं के प्रति प्रतिरोधी बैक्टीरिया पर लागू होता है। मल्टीड्रग-रेसिस्टेंट (एमडीआर) बैक्टीरिया एक से अधिक एंटीबायोटिक के लिए प्रतिरोधी बन जाते हैं। यह दो अलग-अलग तरीकों से हो सकता है - (1) एक जीवाणु में कई अलग-अलग प्रतिरोध जीन हो सकते हैं, प्रत्येक एक विशेष एंटीबायोटिक के लिए प्रतिरोध प्रदान करता है, या (2) एक एकल प्रतिरोध तंत्र एक से अधिक एंटीबायोटिक के लिए प्रतिरोध देता है। दिए गए कथनों में से 3 निश्चित रूप से सही है। इसलिए, विकल्प (ए) से इंकार किया जाता है। एएमआर या एमडीआर किसी भी रोगी की व्यक्तिगत संरचना के कारण नहीं है, लेकिन सिस्टम की विफलता (अत्यधिक या सही नही उपयोग) के कारण है।


49. प्रायः समाचारों में आने वाला Cas9 प्रोटीन क्या है?

  1. लक्ष्य-साधित जीन संपादन (टारगेटेड जीन एडिटिंग) में प्रयुक्त आण्विक कैंची
  2. रोगियों में रोगजनकों की ठीक-ठीक पहचान के लिए प्रयुक्त जैव संवेदक
  3. एक जीन जो पादपों को पीड़क-प्रतिरोधी बनाता है
  4. आनुवंशिकतः रूपांतरित फसलों में संश्लेषित होने वाला एक शाकनाशी पदार्थ

उत्तर (ए) सीएएस 9 (सीआरआईएसपीआर संबद्ध प्रोटीन 9) का उपयोग आनुवंशिक इंजीनियरिंग अनुप्रयोगों में व्यापक रूप से किया जाता है। इसका मुख्य कार्य डीएनए में कटौती करना है एवं इसके लिए यह कोशिका के जीनोम को बदल सकता है। विभिन्न प्रकार के चुनौतीपूर्ण, अमानवीय आवासों में जीवित रहने के लिए, जो बैक्टीरियोफेज से भरे होते हैं, बैक्टीरिया एवं तीरंदाजी ने शिकारी वायरस से बचने एवं बंद करने के लिए उन्नत तरीके विकसित किए हैं। इसमें अनुकूली प्रतिरक्षा की सीआरआईएसपीआर प्रणाली शामिल है। सीएएस 9 अन्य बैक्टीरिया के बीच स्ट्रेप्टोकोकस पाइजेन्स में सीआरआईएसपीआर (क्लस्टर्ड रेगुलरली इंटर्सेस्ड शॉर्ट पैलिंड्रोमिक रिपीट) से जुड़ा एक आरएनए-निर्देशित डीएनए एंडोन्यूक्लिज एंजाइम है। संदर्भ छवि यहां है।


50.निम्नलिखित में से कौन-सा एक कथन सही नहीं है?

  1. यकृतशोथ B विषाणु काफी कुछ HIV की तरह ही संचारित होता है
  2. यकृतशोथ C का टीका होता है, जबकि यकृतशोथ B का कोई टीका नहीं होता
  3. सार्वभौम रूप से यकृतशोथ B और C विषाणुओं से संक्रमित व्यक्तियों की संख्या HIV से संक्रमित लोगों की संख्या से कई गुना अधिक है
  4. यकृतशोथ B और C विषाणुओं से संक्रमित कुल व्यक्तियों में अनेक वर्षों तक इसके लक्षण दिखाई नहीं देते ।

उत्तर (बी) वैक्सीन हेपेटाइटिस बी के लिए उपलब्ध है, न कि हेपेटाइटिस सी। इसलिए (बी) सही नही है, एवं यह हमारा जवाब है। संदर्भ छवि यहां है।


download scanned papers here - Paper 1 Paper II


51.मुग़ल भारत के संदर्भ में, जागीरदार और जमींदार के बीच क्या अंतर है/हैं?

  1. जागीरदारों के पास न्यायिक और पुलिस दायित्वों के एवज में भूमि आबंटनों का अधिकार होता था, जबकि जमींदारों के पास राजस्व अधिकार होते थे तथा उन पर राजस्व उगाही को छोड़कर अन्य कोई दायित्व पूरा करने की बाध्यता नहीं होती थी।
  2. जागीरदारों को किए गए भूमि आबंटन वंशानुगत होते थे और जमींदारों के राजस्व अधिकार वंशानुगत नहीं होते थे।

नीचे दिए गए कूट का प्रयोग कर सही उत्तर चुनिए।

  1. केवल 1
  2. केवल 2
  3. 1 और 2 दोनों
  4. न तो 1, न ही 2

उत्तर (ए) जागीरदार एवं जमींदार दो प्रकार के बिचैलिये थे जिन्होंने मुगल काल के दौरान राजा के लिए भू-राजस्व एकत्र किया था। जागीरदार - वे राजा के अधिकारी थे जो राजा से भेंट में प्राप्त भूमि का आनंद लेते थे। उन्हें मनसबदार कहा जाता था। उन्हें गैर-विरासत योग्य भूमि क्षेत्र आवंटित किया जाता था जो उनकी शुल्क राशि के बराबर होता था जिसे जगीर कहा जाता था। अतः कथन 2 सही नही है। उत्तर केवल (ए) या (डी) हो सकता है। आवंटित किए गए मंसब प्रत्येक वर्ष के लिए समान नहीं रहते थे एवं समय-समय पर संशोधित किए जाते थे यानी जागीरदार को पदोन्नति या पदोवनति होती थी। उनके अधीन भूमि क्षेत्र अधिक होने पर जागीरदार अधिक शक्तिशाली होते थे एवं पुलिस क्षेत्राधिकार का भी आनंद लेते थे। अकबर के शासनकाल के दौरान मनबसबदारी प्रणाली (सामंती प्रणाली का भारतीय संस्करण) के आगमन के बाद जागीरदार का कार्यालय अस्तित्व में आया (हालांकि इसी प्रकार के अधिकारी दिल्ली-सल्तनत काल में भी मौजूद थे)। जागीरदारों को सैनिकों के दल दिए जाते थे एवं उनके वेतन एवं अन्य जरूरतों के लिए वे जिम्मेदार थे। जब कोई जागीरदार आवंटित जागीर पर संप्रभुता का दावा करता था या अन्यथा वैसे भी उन्हें प्रायः प्रति 3-4 वर्ष बाद नियमित रूप से स्थानांतरित कर दिया जाता था। जमींदार - जमीन का धारक जमींदार था। भूमि राजस्व के अलावा किसान के उत्पादन में हिस्सेदारी का दावा करने के लिए उनके पास भूमि पर वंशानुगत अधिकार था। अधिकांश मुगल काल में जमींदारों ने जमीन के वास्तविक बुआईकर्ता से भू-राजस्व एकत्र किया एवं इसके लिए उन्हें फसल का दसवां हिस्सा या तो नगद या करमुक्त भूमि के रूप में दिया जाता था। इस प्रकार, मुगल काल के दौरान जमींदार एवं किसानों के कब्जे वाले गाँव अगल-बगल पाए गए। राजस्व राज्य द्वारा तय किया जाता था लेकिन किसानों द्वारा देय राशि, जागीरदारों एवं जमींदारों के विवेक पर निर्भर करती थी।


52.स्वतंत्र भारत में भूमि सुधारों के संदर्भ में, निम्नलिखित में से कौन-सा कथन सही है?

  1. हदबंदी कानून पारिवारिक जोत पर केंद्रित थे, न कि व्यक्तिगत जोत पर।
  2. भूमि सुधारों का प्रमुख उद्देश्य सभी भूमिहीनों को कृषि भूमि प्रदान करना था।
  3. इसके परिणामस्वरूप नकदी फसलों की खेती, कृषि का प्रमुख रूप बन गई।
  4. भूमि सुधारों ने हदबंदी सीमाओं को किसी भी प्रकार की छूट की अनुमति नहीं दी।

उत्तर (बी) भारत में भूमि सुधारों के तहत उठाया गया महत्वपूर्ण कदम हदबंदी सीमा को लागू करना था। 1972 तक के पहले चरण में, भूमि सुधार इस बात पर केंद्रित थे कि किसी एक व्यक्ति के पास कुल कितनी जमीन हो सकती है। बाद में, ‘एक परिवार’ को इकाई बनाया गया। विकल्प (ए) सही नही है। कानून ने हदबंदी को लागू किया, एवं सरकार को अधिशेष भूमि पर कब्जा करने की अनुमति दी। ऐसी भूमि तब भूमिहीन किसानों या छोटे किसानों के बीच वितरित की जाती थी। इस प्रकार की हदबंदी लगाने का उद्धेश्य जमीन का केंद्रीयकरण कुछ लोगों के हाथ में जाने से रोकना था। विकल्प (बी) सही है। विकल्प (सी) सही नही है। विकल्प (डी) सही नही है, क्योंकि विभिन्न क्षेत्रों, जैसे शिक्षा, आईटी, वृक्षारोपण आदि को छूट दी गई थी।


53.वैश्विक प्रतियोगित्व रिपोर्ट (ग्लोबल कम्पिटिटिबनेस रिपोर्ट) कौन प्रकाशित करता है?

  1. अंतर्राष्ट्रीय मुद्रा कोष
  2. संयुक्त राष्ट्र व्यापार एवं विकास सम्मेलन (सुनाइटेड नेशंस कॉन्फरेन्स ऑन ट्रेड ऐंड डेवलप्मेंट)
  3. विश्व आर्थिक मंच (वल्ड इकनॉमिक फोरम)
  4. विश्व बैंक

उत्तर (सी) वर्ल्ड इकोनॉमिक फोरम (डब्ल्युईएफ) सालाना रिपोर्ट प्रकाशित करता है। एक सीधा एवं सरल प्रश्न। यह 2004 में पहली बार प्रकाशित हुआ था। यह वैश्विक प्रतिस्पर्धात्मकता सूचकांक, जिसे जेवियर साला-ए-मार्टिन एवं एल्सा वी. अर्तदी द्वारा विकसित किया गया है, के आधार पर देशों को सूचिबद्ध करता है, । रिपोर्ट ‘देशों की उनके नागरिकों को उच्च स्तर की समृद्धि प्रदान करने की क्षमता का आकलन करती है’। यह बदले में इस बात पर निर्भर करता है कि कोई देश उपलब्ध संसाधनों का उपयोग कैसे करता है। इसलिए, वैश्विक प्रतिस्पर्धात्मकता सूचकांक उन संस्थानों, नीतियों एवं कारकों के सेट को मापता है जो आर्थिक समृद्धि के स्थायी वर्तमान एवं मध्यम अवधि के स्तर को निर्धारित करते हैं। संदर्भ छवियाँ यहां है।


54.’चार्टर ऐक्ट, 1813’ के संबंध में, निम्नलिखित कथनों पर विचार कीजिए :

  1. इसने भारत में ईस्ट इंडिया कम्पनी के व्यापार एकाधिपत्य को, चाय का व्यापार तथा चीन के साथ व्यापार को छोड़कर, समाप्त कर दिया ।
  2. इसने कम्पनी द्वारा अधिकार में लिए गए भारतीय राज्यक्षेत्रों पर ब्रिटिश राज (क्राउन) की सम्प्रभुता को सुदृढ़ कर दिया।
  3. भारत का राजस्व अब ब्रिटिश संसद के नियंत्रण में आ गया था।

उपर्युक्त में से कौन-से कथन सही हैं?

  1. केवल 1 और 2
  2. केवल 2 और 3
  3. केवल 1 और 3
  4. 1, 2 और 3

उत्तर (ए) ईस्ट इंडिया कंपनी अधिनियम 1813, जिसे चार्टर एक्ट 1813 भी कहा जाता है, यूनाइटेड किंगडम की संसद का एक अधिनियम था, जिसने ब्रिटिश ईस्ट इंडिया कंपनी को जारी किए गए चार्टर को नवीनीकृत किया एवं भारत में कंपनी के शासन को जारी रखा। हालांकि इस अधिनियम ने, चाय एवं अफीम के व्यापार तथा चीन के साथ व्यापार को छोड़कर, कंपनी की वाणिज्यिक एकाधिकार को समाप्त कर दिया, इस प्रकार यह अधिनियम भारत में ब्रिटिश शक्ति के विकास को दर्शाता है।

1. इस अधिनियम ने ब्रिटिश भारत पर क्राउन की संप्रभुता को स्पष्ट रूप से सुदृढ़ किया।

2. इसने भारतीय जनमानस में शिक्षा को बढ़ावा देने के लिए 100,000 रुपये का आवंटन किया।

3. इस अधिनियम ने ईसाई मिशनरियों को अंग्रेजी का प्रचार करने एवं उनके धर्म का प्रचार करने की अनुमति दी।

यूरोपीय ब्रिटिश विषयों पर भारत में प्रांतीय सरकारों एवं अदालतों की शक्ति को भी अधिनियम द्वारा मजबूत किया गया था। भारतीय साहित्य के पुनरुत्थान को प्रोत्साहित करने एवं विज्ञान के प्रचार के लिए वित्तीय प्रावधान भी किए गए थे।

इसलिए, केवल 1 एवं 2 सत्य हैं। भारत पर ब्रिटिश संसद ने 1857 के बाद ही पूर्ण नियंत्रण लिया।


55.स्वदेशी आंदोलन के संदर्भ में, निम्नलिखित कथनों विचार कीजिए :

  1. इसने देशी शिल्पकारों के कौशल तथा उद्योगों को पुनर्जीवित करने में योगदान किया।
  2. स्वदेशी आंदोलन के एक अवयव के रूप में राष्ट्रीय शिक्षा परिषद् की स्थापना हुई थी।

उपर्युक्त में से कौन-सा/से कथन सही है/हैं?

  1. केवल 1
  2. केवल 2
  3. 1 और 2 दोनों
  4. न तो 1, न ही 2

उत्तर (सी) 1 एवं 2 दोनों सही हैं। स्वदेशी आंदोलन का एक महत्वपूर्ण पहलू आत्मनिर्भरता या आत्मशक्ति (रवीन्द्र नाथ टैगोर के अनुसार) पर जोर था। कई विशिष्ट भारतीय औद्योगीक उपक्रम, बंगाल केमिकल स्वदेशी स्टोर्स (आचार्य पीसी रे द्वारा खोला गया), लक्ष्मी कॉटन मिल्स, मोहिनी मिल्स एवं नेशनल टेनरी इस दौरान शुरू किए गए थे। अतः कथन (1) सत्य है। राष्ट्रीय शिक्षा परिषद (या एनसीई) 1906 में बंगाल में भारतीय राष्ट्रवादियों द्वारा स्थापित एक संगठन था, जो एक स्वदेशी औद्योगीकरण आंदोलन के हिस्से के रूप में विज्ञान एवं प्रौद्योगिकी को बढ़ावा देने के लिए था। इसने बंगाल नेशनल कॉलेज एवं बंगाल संस्थान की स्थापना की जो बाद में जादवपुर विश्वविद्यालय के रूप में विलय हो गया। परिषद के तहत काम करने वाले संस्थानों को स्वदेशी गतिविधियों के केंद्र के रूप में माना जाता था एवं सरकार ने देशभक्ति गीतों के गायन जैसे राष्ट्रवादी गतिविधियों पर प्रतिबंध लगा दिया था। कथन (2) भी सत्य है।


56.निम्नलिखित युग्मों पर विचार कीजिए :

उपर्युक्त में से कौन-सा/से युग्म सही सुमेलित है/हैं?

  1. केवल 1
  2. केवल 1 और 2
  3. केवल 2 और 3
  4. 1, 2 और 3

उत्तर (डी) अखिल भारतीय अस्पृश्यता विरोधी लीग की स्थापना 1932 में महात्मा गांधी द्वारा की गई थी। अखिल भारतीय किसान सभा की स्थापना स्वामी सहजानंद सरस्वती ने 1936 में की थी। स्व-सम्मान आंदोलन की स्थापना 1921 में एस. रामनाथन ने की थी एवं ई.वी. रामास्वामी को इस अभियान का नेतृत्व करने के लिए आमंत्रित किया था। ई. वी. रामास्वामी (पेरियार के नाम से लोकप्रिय) को ‘आधुनिक तमिलनाडु का जनक’माना जाता है। इसलिए सभी कथन सत्य हैं।


57.निम्नलिखित में से कौन-सा एक हड़प्पा स्थल नहीं है?

  1. चन्हुदड़ो
  2. कोट दीजी
  3. सोहगौरा
  4. देसलपुर

उत्तर (सी) सोहगौरा हड़प्पा सभ्यता का स्थल नहीं है। यह उत्तर प्रदेश राज्य के दक्षिण-पूर्व में गोरखपुर जिले से लगभग 20 किमी दूर राप्ती नदी के तट पर स्थित एक गाँव है, जहाँ ब्राह्मी लिपि में प्राकृत भाषा में लिखे गए ताम्रपत्र शिलालेख की खोज की गई थी। पहले शिलालेख को के अशोक के काल के पूर्व, यहां तक कि मौर्य काल से भी पूर्व का समझा गया, लेकिन प्लेट के लेखन, विशेष रूप से अक्षरों के विन्यास के बाद इसकी तारिख अशोक के बाद की सुझाई गई। अन्य तीन - चान्दू दारो, कोट दीजी एवं देसलपुर प्रसिद्ध सिंधु घाटी सभ्यता स्थल हैं। संदर्भ छवि यहां है।


58.निम्नलिखित में से किस उभारदार मूर्तिशिल्प (रिलीफ स्कल्प्चर) शिलालेख में अशोक के प्रस्तर रूपचित्र के साथ ’राण्यो अशोक’ (राजा अशोक) उल्लिखित है?

  1. कंगनहल्ली
  2. साँची
  3. शाहबाजगढ़ी
  4. सोहगौरा

उत्तर (ए) कर्नाटक के कंगनाहल्ली के अल्पज्ञात गाँव के पास, एक असाधारण बौद्ध स्तूप के अवशेष हैं। उस के बाद आज, कंगनाहल्ली गाँव को 1954 में एक बौद्ध स्थल के रूप में फिर से खोजा गया एवं 1990 के दशक में भारतीय पुरातत्व सर्वेक्षण (एएसआई) द्वारा खुदाई की गई। उदात्त पैनलों पर बुद्ध की माता माया के सपने से लेकर उनके ज्ञानोदय तक बुद्ध के जीवन का वर्णन किया गया हैं। यक्ष एवं नागों के प्रतिनिधियों के साथ मौर्य सम्राट अशोक का चित्रण करने वाले पहले नामित पट्टिका इसके खजाने में हैं। एवं इसके अलावा, जानवरों की एक विलक्षण श्रृंखला है - पंखों वाले शेर, हाथी, ऊंट, हिरण एवं बैल - एक दिशा में चलते हुए, मानो वे वहाँ के पवित्र चौत्य की परिक्रमा कर रहे हों। संदर्भ छवि यहां है।


59.निम्नलिखित पर विचार कीजिए :

  1. बुद्ध में देवत्वारोपण
  2. बोधिसत्व के पथ पर चलना
  3. मूर्ति उपासना तथा अनुष्ठान

उपर्युक्त में से कौन-सी विशेषता/विशेषताएँ महायान बौद्धमत की है/हैं?

  1. केवल 1
  2. केवल 1 और 2
  3. केवल 2 और 3
  4. 1, 2 और 3

उत्तर (बी) महायान बौद्ध धर्म की कुछ प्रमुख विशेषताओं पर विचार करें। 1 एवं 2 वास्तव में सच हैं, लेकिन 3 नहीं है। यद्यपि बौद्धों के बीच बुद्ध की छवियों को रखने एवं बुद्ध के प्रति अपने सम्मान का भुगतान करने की प्रथा है, किंतु फिर भी बौद्धों को मूर्तिपूजक नहीं कहा जा सकता हैं। बौद्ध सबसे महान, बुद्धिमान, सबसे दयालु, एवं पवित्र व्यक्ति के रूप में बुद्ध की छवि का सम्मान करते हैं जो कभी इस दुनिया में रहे हैं। छवि एक दृश्य सहायता है जो मन में बुद्ध को याद करने एवं उनके महान गुणों को याद करने में मदद करती है जिसने लाखों लोगों को पीढ़ी-दर-पीढ़ी सभ्य दुनिया में प्रेरित किया। इसलिए बुद्ध को देवता की तरह माना गया है, लेकिन उनकी पूजा नहीं की गई है। बोधिसत्व एक प्रबुद्धता है जो सभी संतानों को आत्मज्ञान प्राप्त करने में मदद करने के लिए संस्कार (किसी भी स्तर) में रहने की प्रतिज्ञा कराता है, विशेषतः ज्ञान एवं करुणा द्वारा।


60.गुप्त काल के दौरान भारत में बलात् श्रम (विष्टि) के संदर्भ में, निम्नलिखित में से कौन-सा कथन सही है?

  1. इसे राज्य के लिए आय का एक स्त्रोत, जनता द्वारा दिया जाने वाला एक प्रकार का कर, माना जाता था
  2. यह गुप्त साम्राज्य के मध्य प्रदेश तथा काठियावाड़ क्षेत्रों में पूर्णतः अविद्यमान था।
  3. बलात् श्रमिक साप्ताहिक मजदूरी का हकदार होता था।
  4. मजदूर के ज्येष्ठ पुत्र को बलात् श्रमिक के रूप में में भेज दिया जाता था।

उत्तर (ए) ‘विष्टि’ का अर्थ है जबरन मजदूरी कराना, इसमें मजदूरों को उनके मजदूरी का भुगतान नहीं किया जाता है। गुप्त काल में, मध्य एवं पश्चिमी भारत में, ग्रामीणों को शाही सेना एवं अधिकारियों द्वारा इसके अधीन किया गया था। जबरन श्रम (विष्टी) लगाने एवं कई नए लगान एवं करों के कारण किसानों की स्थिति गुप्त काल से ही खराब होना शुरू हो गई थी। अपने क्षेत्र से गुजरने पर किसानों को शाही सेना को खिलाना पड़ता था। शाही सेना के लिए भी विष्टि कराया जाता था।


61.निम्नलिखित में से कौन-सा एक पादप-समूह ’नवीन विश्व (न्यू वर्ल्ड)’ में कृषि-योग्य बनाया गया तथा इसका ’प्राचीन विश्व (ओल्ड वर्ल्ड)’ में प्रचलन शुरू किया गया?

  1. तंबाकू, कोको और रबड़
  2. तंबाकू, कपास और रबड़
  3. कपास, कॉफी और गन्ना
  4. रबड़, कॉफी और गेहूँ

उत्तर (ए) नई दुनिया की फसलें वे थीं जो केवल अमेरिका (उत्तर एवं दक्षिण) में उगाई जाती थीं, जब यूरोपीय वहां पहुंचे, तो वे इन फसलों को अपने साथ ले आए। विकल्प (ए) सही है क्योंकि तंबाकू, कोको एवं रबर नई दुनिया की फसलें हैं। विकल्प (बी) में, कपास का उल्लेख किया गया है जो निश्चित रूप से नई दुनिया में उगाया गया था, लेकिन पुरानी दुनिया में भी। भारतीय उपमहाद्वीप में कपास के उपयोग के सबसे पुराने साक्ष्य मेहरगढ़ एवं राखीगढ़ी में पाए गए हैं जहाँ कपास के धागे तांबे के मोतियों में संरक्षित पाए गए हैं। इन खोजों को नवपाषाण (5 वीं सहस्त्राब्दी ईसा पूर्व) में दिनांकित किया गया है। इसलिए विकल्प (बी) एवं (सी) सही नही हैं। गेहूं एक पुरानी दुनिया की फसल थी। आम पुरानी दुनिया की फसलें (जैसे, जौ, मसूर, जई, मटर, राई, गेहूं) एवं पालतू जानवर (जैसे, मवेशी, मुर्गियां, बकरी, घोड़े, सूअर, भेड़) तब तक अमेरिका में मौजूद नहीं थे, जब तक कि 1490 के दशक में पोस्ट-कोलम्बियन संपर्क में नही आए (‘कोलंबियन एक्सचेंज’ के हिस्से के रूप में)। अतः, (डी) सही नही है। संदर्भ छवि यहां है।


62.निम्नलिखित कथनों पर विचार कीजिए :

  1. एशियाई शेर प्राकृतिक रूप से सिर्फ भारत में पाया जाता है।
  2. दो-कूबड़ वाला ऊँट प्राकृतिक रूप से सिर्फ भारत में पाया जाता है।
  3. एक-सींग वाला गैंडा प्राकृतिक रूप से सिर्फ भारत में पाया जाता है।

उपर्युक्त में से कौन-सा/से कथन सही है/हैं?

  1. तंबाकू, कोको और रबड़
  2. केवल 2
  3. केवल 1 और 3
  4. 1, 2 और 3

उत्तर (ए) एशियाई शेर अब केवल गिर अभयारण्य, गुजरात, भारत में पाया जाता है। अतः 1 सही है। अतः, विकल्प (बी) सही नही है। डबल-कूबड़ वाला ऊंट - बैक्ट्रियन ऊंट (कैमलस बैक्टिरियनस) - एक बड़ा, जुड़े हुए पंजों वाला प्राणी है जो प्राकृतिक रूप से मध्य एशिया का रहवासी है । यह भारत में भी पाया जाता है। अतः, 2 सही नही है (‘केवल’)। तो, विकल्प (बी) एवं (डी) सही नही हैं। एक सींग वाला गैंडा नेपाल में भी पाया जाता है, इसलिए 3 सही नही है। तो, उत्तर (ए) है ।


63.निम्नलिखित युग्मों पर विचार कीजिए :

उपर्युक्त में से कौन-से युग्म सही समुलित हैं?

  1. केवल 1 और 2
  2. केवल 2 और 3
  3. केवल 1 और 3
  4. 1, 2 और 3

उत्तर (ए) चंद्रभागा महाराष्ट्र के अमरावती जिले की प्रमुख नदियों में से एक है। पूर्णा (भीम) की सहायक नदी के रूप में, यह ताप्ती-पूर्ण नदी प्रणाली का एक हिस्सा है। नदी को चंद्रभागा नदी के रूप में संदर्भित किया जाता है, विशेष रूप से पंढरपुर में, क्योंकि यह चंद्रमा के आकार के समान है। तिरुचिरापल्ली से निकलने वाली दो प्रमुख नदियाँ कावेरी एवं उसकी सहायक नदी कोल्लीदाम हैं, लेकिन यह शहर उईयाकोंडान चैनल का प्रवाह भी है। हम्पी तुंगभद्रा नदी के तट पर स्थित है, न कि मलप्रभा नदी। धारवाड़ जिले के मालप्रभा में बेनीहिल्ला, हीराला एवं तुपरिहल्ला प्रमुख सहायक नदियाँ हैं।


64.किसी दिए गए वर्ष में भारत में कुछ राज्यों में आधिकारिक गरीबी रेखाएँ अन्य राज्यों की तुलना में उच्चतर हैं, क्योंकि

  1. गरीबी की दर अलग-अलग राज्य में अलग-अलग होती है
  2. कीमत-स्तर अलग-अलग राज्य में अलग-अलग होता है
  3. सकल राज्य उत्पाद अलग-अलग राज्य में अलग-अलग होता है
  4. सार्वजनिक वितरण की गुणता अलग-अलग राज्य में अलग-अलग होती है।

उत्तर (बी) विकल्प (ए) एक सही स्पष्टीकरण नही है। गरीबी दर गरीबी रेखा निर्धारित नहीं कर सकती, लेकिन गरीबी रेखा गरीबी दर निर्धारित करेगी। विकल्प (बी) सही है। खपत का स्तर मूल्य स्तरों पर निर्भर करता हैं, एवं तदनुसार गरीबी रेखाएं अलग-अलग परिभाषित की जाती हैं। विकल्प (सी) सही नही है क्योंकि जीएसडीपी पर गरीबी रेखा तय करते समय ध्यान नहीं दिया जाता है। विकल्प (डी) सही नहीं है क्योंकि, पीडीएस गुणवत्ता गरीबी को प्रभावित कर सकती है लेकिन इसका उपयोग गरीबी रेखा की परिभाषित करने में नहीं किया जाता है। संदर्भ छवि यहां है।


65.निम्नलिखित में से किसके संदर्भ में, कुछ वैज्ञानिक पक्षाभ मेघ विरलन तकनीक तथा समतापमंडल में सल्फेट वायुविलय अंतःक्षेपण के उपयोग का सुझाव देते हैं?

  1. कुछ क्षेत्रों में कृत्रिम वर्षा करवाने के लिए
  2. उष्णकटिबंधी चक्रवातों की बारंबारता और तीव्रता को कम करने के लिए
  3. पृथ्वी पर सौर पवनों के प्रतिकूल प्रभाव को कम करने के लिए
  4. भूमंडलीय तापन को कम करने के लिए

उत्तर (डी) स्ट्रेटोस्फीयर (समताप मंड़ल) में सल्फेट के इंजेक्शन के माध्यम से पृथ्वी को ठंडा करना सबसे अधिक चर्चा में आई भू-इंजीनियरिंग (जीई) योजनाओं में से एक है। स्ट्रैटोस्फेरिक एरोसोल ट्रोपोस्फीयर में तलछट कर सकते हैं, एरोसोल रचना को संशोधित कर सकते हैं एवं इस प्रकार सिरस बादलों को प्रभावित कर सकते हैं। यह अनुमान है कि कई जीई दृष्टिकोणों द्वारा प्रस्तावित स्ट्रैटोस्फेरिक एरोसोल लोडिंग में वृद्धि होने से बर्फ के न्यूक्लियोनियन दर में कमी आएगी एवं इस प्रकार वैकल्पिक रूप से पतले सिरस बादल बनेंगे। ये वैकल्पिक रूप से पतले सिरस के बादल लंबी-लहर में एक मजबूत नकारात्मक बादल को उत्पन्न करते हैं, जो समग्र शुद्ध जीई फोर्सिंग में 60 फीसदी का योगदान देता है। कुल मिलाकर वैश्विक उष्मन को कम किया जा सकता है।


66.निम्नलिखित में से किसके संदर्भ में, ’ताप-अपघटन और प्लाज़्मा गैसीकरण’ शब्दों का उल्लेख किया गया है?

  1. दुर्भभ (रेअर) भू-तत्वों का निष्कर्षण
  2. प्राकृतिक गैस निष्कर्षण प्रौद्योगिकी
  3. हाइड्रोजन ईंधन-आधारित ऑटोमोबाइल
  4. अपशिष्ट-से-ऊर्जा प्रौद्योगिकी

उत्तर (डी) पायरोलिसिस प्रक्रियाओं का उपयोग करने वाले कारखानों के खराब प्रबंधन के कारण पर्यावरण प्रदूषण के कारण होने वाले के विवाद हाल ही में सुर्खियों में रहे हैं। पायरोलिसिस एक निष्क्रिय वातावरण में ऊंचे तापमान पर प्रदार्थों का थर्मल अपघटन है। ऑक्सीजन के बिना दहन के माध्यम से अपशिष्ट प्लास्टिक (टायर आदि) को तेल में बदलने के लिए पायरोलिसिस प्रक्रिया का उपयोग किया जाता है। पायरोलिसिस का उपयोग प्राचीन काल से लकड़ी का कोयला बनाने में किया जाता रहा है। इसी तरह, प्लाज्मा गैसीकरण, प्लाज्मा का उपयोग करके एक चरम थर्मल प्रक्रिया है जो कार्बनिक पदार्थों को एक सिग्नस में परिवर्तित करता है जो मुख्य रूप से हाइड्रोजन एवं कार्बन मोनोऑक्साइड से बना है। एक विद्युत चाप द्वारा संचालित एक प्लाज्मा टॉर्च का उपयोग गैस को आयनित करने एवं कार्बनिक पदार्थों को सिनेगैस में उत्प्रेरित करने के लिए किया जाता है, जिसके साथ स्लैग एक उपोत्पाद के रूप में शेष रहता है। दोनों “अपशिष्ट-से-ऊर्जा प्रौद्योगिकियाँ हैं। संदर्भ छवि यहां हैं।


67.निम्नलिखित में से कौन-से अगस्त्यमाला जीवमंडल रिज़र्व में आते हैं

  1. नेय्यार, पेप्पारा और शेंदुर्ने वन्य प्राणी अभयारण्य; और कलाकड़ मुंदन्थुराई बाघ रिज़र्व
  2. मुदुमलाई, सत्यमंगलम और वायनाड वन्य प्राणी अभ्यारण्य; और साइलेंट वैली नैशनल पार्क
  3. कैंडिन्य, गुंडला ब्रह्मेश्वर और पापीकोंडा वन्य प्राणी अभयारण्य; और मुकुर्थी नैशनल पार्क
  4. कावल और श्रीवेंकटेश्वर वन्य प्राणी अभयारण्य; और नागार्जुनसागर-श्रीशैलम बाघ रिज़र्व

उत्तर (ए) पश्चिमी घाट में भारत का अगस्त्यमाला बायोस्फीयर रिजर्व संयुक्त राष्ट्र के शीर्ष सांस्कृतिक निकाय यूनेस्को द्वारा अपने विश्व नेटवर्क ऑफ बायोस्फीयर रिजर्व में 2016 में जोड़े गए 20 नए स्थलों में से एक था। इसकी स्थापना वर्ष 2001 में की गई थी एवं यह केरल एवं तमिलनाड़ु में फैला हुआ है। एक कठिन प्रश्न, जिसमें आपसे प्रत्येक रिजर्व के विवरण को याद रखने की उम्मीद थी। एक उपयोगी पृष्ठ यहाँ है http://civils.pteducation.com/p/protectedareasinindia.html


68.निम्नलिखित कथनों पर विचार कीजिए :

  1. समुद्री कच्छपों की कुछ जातियाँ शाकभक्षी होती हैं।
  2. मछली की कुछ जातियाँ शाकभक्षी होती हैं।
  3. समुद्री स्तनपायियों की कुछ जातियाँ शाकभक्षी होती हैं।
  4. साँपो की कुछ जातियाँ सजीवप्रजक होती हैं।

उपर्युक्त में से कौन-से कथन सही हैं ?

  1. केवल 1 और 3
  2. केवल 2, 3 और 4
  3. केवल 2 और 4
  4. 1, 2, 3 और 4

उत्तर (डी) हम निश्चित रूप से जानते हैं कि कुछ समुद्री स्तनधारी शाकाहारी हैं (जैसे समुद्री गाय - मानेटी)। अतः कथन 3 सही है, एवं विकल्प (सी) सही नही है। बोआ कंस्ट्रक्टर एवं ग्रीन एनाकोंडा, विविपेरस सांप के दो उदाहरण हैं, जिसका अर्थ है अंडों की मदद के बिना बच्चों को जन्म देते हैं। अतः कथन 4 सही है। अतः, विकल्प (ए) भी सही नही है। अब कथन 1 को जांचें 1, कुछ कछुए वास्तव में शाकाहारी हैं। इसलिए, विकल्प (डी) सही है।


69.निम्नलिखित युग्मों पर विचार कीजिए :

उपर्युक्त में से कौन-से युग्म सही समुलित हैं?

  1. केवल 1 और 2
  2. केवल 2 और 3
  3. केवल 1 और 3
  4. 1, 2 और 3

उत्तर (सी) सबसे बड़ा सुराग ‘इरावडी डॉल्फिन’ था। यह चंबल नदी में नहीं पाया जा सकता है, यह केवल भारत के पूर्वी हिस्से में या मुख्य इरावदी नदी में (म्यांमार) में पाया जाता है। अतः कथन 2 सही नही है। तो, विकल्प (सी) सही है! रस्टी-स्पॉटेड बिल्ली सबसे छोटी जंगली बिल्ली है। संदर्भ छवि यहां हैं।


70.पर्यावरण में निर्मुक्त हो जाने वाली ’सूक्ष्ममणिकाओं (माइक्रोबीड्स)’ के विषय में अत्यधिक चिंता क्यों है?

  1. ये समुद्री परितंत्रो के लिए हानिकारक मानी जाती है।
  2. ये बच्चों में त्वचा कैंसर होने का कारण मानी जाती हैं।
  3. ये इतनी छोटी होती है कि सिंचित क्षेत्रों में फसल पादपों द्वारा अवशोषित हो जाती है।
  4. अक्सर इनका इस्तेमाल खाद्य-पदार्थों में मिलावट के लिए किया जाता है।

उत्तर (ए)) सूक्ष्मकण अपने सबसे बड़े आयाम में एक मिलीमीटर से कम के ठोस प्लास्टिक कण होते हैं वे अक्सर पॉलीइथाइलीन से बने होते हैं, लेकिन पॉलीप्रोपाइलीन एवं पॉलीस्टीरिन जैसे अन्य पेट्रोकेमिकल प्लास्टिक के भी हो सकते हैं। पिछले कुछ वर्षों में इन ‘माइक्रोप्लास्टिक्स’के समुद्री पारिस्थितिकी तथा समुद्री जानवरों के मृत शरीरों में पाए जाने से अनेक विवाद एवं चिंता पैदा हुई है।



download scanned papers here - Paper 1 Paper II


71.किसके राज्य में ’कल्याण मंडप’ की रचना मंदिर-निर्माण का एक विशिष्ट अभिलक्षण था?

  1. चालुक्य
  2. चंदेल
  3. राष्ट्रकूट
  4. विजयनगर

उत्तर (डी) विजयनगर मंदिर आमतौर पर एक मजबूत बाड़े से घिरे होते हैं। मध्यम आकार के मंदिरों में एक गर्भगृह, शुकसैनी (एंटिचेम्बर), एक नवरंगा (अंतराला) जो कि गर्भगृह एवं बाहरी मंडप (हॉल) एवं एक रंगमंतापा (संलग्न स्तंभ हॉल) को जोड़ता है। इन संरचनाओं के अलावा, मध्यम आकार के मंदिरों में गर्भगृह के चारों ओर एक प्रदक्षिणा पथ, एक खुला महामंतापा (बड़ा हॉल), एक कल्याण मंडप (औपचारिक हॉल) एवं एक मंदिर की टंकी है, जो वार्षिक उत्सव की जरूरतों को पूरा करती है।


72.निम्नलिखित कथनों पर विचार कीजिए :

  1. दिल्ली सल्तनत के राजस्व प्रशासन में राजस्व वसूली के प्रभारी को ’आमिल’ कहा जाता था।
  2. दिल्ली के सुल्तानों की इक्ता प्रणाली एक प्राचीन देशी संस्था थी।
  3. ’मीर बख्शी का पद दिल्ली के ख़लजी सुल्तानों के शासनकाल में अस्तित्व में आया।

उपर्युक्त में से कौन-सा/से कथन सही है/हैं?

  1. केवल 1
  2. केवल 1 और 2
  3. केवल 3
  4. 1, 2 और 3

उत्तर (ए) दिल्ली सल्तनत में, एक परगना के महत्वपूर्ण अधिकारी अमिल, मुश्रीफ (अमीन या मुंसिफ खजांची के नाम से भी जाने जाते थे), कानुनगो एवं दो कारकुन (क्लर्क) थे। केंद्र सरकार ने प्रत्येक उप-मंडल में ‘अमिल’ या ‘राजस्व-कलेक्टर’ नियुक्त किया, जिसे ‘शिक’ कहा जाता है। वे गाँव के वंशानुगत अधिकारियों जैसे चौधरियों, मुक्कदम्स, पटवारियों आदि की सहायता से राजस्व एकत्र करते थे, अतः कथन (1) सत्य है। दिल्ली की सल्तनत में इल्तुतमिश द्वारा सबसे पहले इक्ता प्रणाली की शुरुआत की गई थी। इसलिए कथन (2) सत्य नहीं है (यह एक प्राचीन स्थानीय संस्थान नहीं था)। ‘मीर बख्शी’ का कार्यालय मुगल बादशाह अकबर द्वारा शुरू किया गया था। अतः कथन (3) भी सत्य नहीं है।


73.निम्नलिखित कथनों पर विचार कीजिए :

  1. संत निम्बार्क, अकबर के समकालीन थे।
  2. संत कबीर, शेख अहमद सरहिंदी के अत्यधिक प्रभावित थे।

उपर्युक्त में से कौन-सा/से कथन सही है/हैं?

  1. केवल 1
  2. केवल 2
  3. 1 और 2 दोनों
  4. न तो 1, न ही 2

उत्तर (डी) संत निम्बार्क एक हिंदू दार्शनिक एवं टिप्पणीकार थे, जिन्हें भेद-अभेद द्वैत-अद्वैत के वैष्णव सिद्धांत का प्रचार करने के लिए जाना जाता है। निम्बार्क के रहने एवं पढ़ाने की तारीखों को लेकर काफी असहमति है। वैदिक शास्त्रों के अनुसार, उनका जन्म 3096 ईसा पूर्व में हुआ था, लेकिन आधुनिक ऐतिहासिक शोध उन्हें तेरहवीं या चौदहवीं शताब्दी में रखते हैं। अतः कथन (1) सत्य नहीं है। कुछ इतिहासकारों का मानना है कि कबीर का जीवन काल 1398 ईस्वी - 1448 ईस्वी है, जबकि अन्य 1440 ईस्वी -1518 ईस्वी के पक्ष में हैं। शेख अहमद अल-सरहिंदी (1564 ईस्वी -1624 ईस्वी) एक इस्लामिक विद्वान, हनाफी न्यायविद, एवं मुगल काल के दौरान नक्शबंदी सूफी आदेश के प्रमुख सदस्य थे। स्पष्ट रूप से कबीर शेख अहमद सरहिंद से प्रभावित नहीं हो सकते। कथन (2) सत्य नहीं है।


74.भारत में ब्रिटिश औपनिवेशिक शासन के संदर्भ में, निम्नलिखित कथनों पर विचार कीजिए :

  1. महात्मा गाँधी ’गिरमिटिया (इंडेंचर्ड लेबर)’ प्रणाली के उन्मुलन में सहायक थे।
  2. लॉर्ड चेम्सफोर्ड की ’वॉर कान््फरेन्स’ में महात्मा गाँधी ने विश्व युद्ध के लिए भारतीयों की भारती से संबंधित प्रस्ताव का समर्थन नहीं किया था।
  3. भारत के लोगों द्वारा नमक कानून तोड़े जाने के परिणामस्वरूप, औपनिवेशिक शासकों द्वारा भारतीय राष्ट्रीय काँग्रेस को अवैध घोषित कर दिया गया था।

उपर्युक्त में से कौन-से कथन सही हैं?

  1. केवल 1 और 2
  2. केवल 1 और 3
  3. केवल 2 और 3
  4. 1, 2 और 3

उत्तर (बी) भारतीय गिरमिटिया प्रणाली या इंडेचर्ड लेबर एक प्रणाली थी जो ‘ऋण-बंधन’का एक रूप था जिसके द्वारा एक मिलियन से अधिक भारतीयों को विशेष रूप से चीनी के कारखानों में श्रम प्रदान करने के लिए यूरोपीय शक्तियों के विभिन्न उपनिवेशों में पहुंचाया गया था। यह 1833 में गुलामी की प्रथा के अंत से शुरू हुआ एवं 1920 तक जारी रहा। महात्मा गांधी ने वास्तव में ‘गिरमिटिया मजदूर’ की व्यवस्था के उन्मूलन में महत्वपूर्ण भूमिका निभाई । उन्होंने पहली बार दक्षिण अफ्रीका में एशियाई गिरमिटिया मजदूरों की दुर्दशा देखी एवं 20 वीं सदी के पहले दशक के दौरान इस मुद्दे पर अभियान चलाया। 1917 में ब्रिटिश सरकार द्वारा गिरमिटिया श्रमिकों की प्रणाली को आधिकारिक रूप से समाप्त कर दिया गया था। इसलिए, 1 सही है। 1916 से 1921 तक भारत के वायसराय लॉर्ड चेम्सफोर्ड ने गांधी को एक ‘वॉर कॉन्फरेन्स’ में दिल्ली आमंत्रित किया। साम्राज्य का विश्वास हासिल करने के लिए, गांधी प्रथम विश्व युद्ध मे भारतीय सैनिकों के भाग लेने की बात से सहमत हुए। हालांकि उन्होंने वायसराय को लिखा एवं कहा कि वह ‘व्यक्तिगत रूप से किसी को भी नहीं मारेंगे या घायल नहीं करेंगे, दोस्त या दुश्मन’। इसलिए कथन (2) सही नहीं है। 1930 में नमक सत्याग्रह के बाद, 1932 में, ब्रिटिश सरकार ने भारतीय राष्ट्रीय कांग्रेस को अवैध घोषित कर दिया एवं गांधीजी को फिर से गिरफ्तार कर लिया। अतः कथन (3) सही है।


75.भारतीय राष्ट्रीय आंदोलन के संदर्भ में, निम्नलिखित युग्मों पर विचार कीजिए :

उपर्युक्त में से कौन-सा/से युग्म सही सुमेलित है/हैं?

  1. केवल 1
  2. केवल 1 और 2
  3. केवल 3
  4. 1, 2 और 3

उत्तर (डी) सब सुमेलित हैं। भारत के संविधान सभा के सदस्य के रूप में क्षितिज चंद्र नियोगी (1888-1970), स्वतंत्र भारत के पहले मंत्रिमंडल के सदस्य एवं भारत के पहले वित्त आयोग के अध्यक्ष थे। अतः कथन (2) सत्य है। पूरन चंद जोशी (14 अप्रैल 1907 - 9 नवंबर 1980), 1935-47 तक भारतीय कम्युनिस्ट पार्टी के पहले महासचिव थे। अतः कथन (3) सत्य है। ‘इंडियन नेशनल लिबरल फेडरेशन’ की स्थापना 1919 में सुरेंद्र नाथ बनारजिया ने की थी एवं इसके कुछ प्रमुख नेता तेज बहादुर सप्रू, वी.एस. श्रीनिवास शास्त्री एवं एम. आर. जयकर थे। तेज बहादुर सप्रू उदारवादियों के बीच सबसे महत्वपूर्ण नेता के रूप में उभरे। अतः कथन (1) भी सत्य है।


76.मियाँ तानसेन के संदर्भ में, निम्नलिखित में से कौन-सा कथन सही नहीं है?

  1. सम्राट अकबर द्वारा इन्हें दी गई उपाधि तानसेन थी।
  2. तानसेन ने हिन्दू देवी-देवताओं से संबंधित धु्रपदों की रचना की।
  3. तानसेन ने अपने संरक्षकों से संबंधित गानों की रचना की।
  4. तानसेन से अनेक रागों की मौलिक रचना की।

उत्तर (ए) तानसेन (1500 इस्वी - 1586 इस्वी), उत्तर भारतीय (हिंदुस्तानी) शास्त्रीय संगीत के एक प्रमुख व्यक्ति थे। तानसेन का वास्तविक नाम रामतनु था। तानसेन ग्वालियर के राजा द्वारा उन्हें दी गई उपाधि थी। तानसेन बांदवगढ़ (रीवा) के राजा रामचंद्र के दरबार में संगीतकार थे। जब अकबर ने उनकी विलक्षण प्रतिभा के बारे में सुना, तो उन्होंने तानसेन को बुलाने के लिए एक ‘फरमान’ भेजा एवं उन्हें अपने दरबार में नवरत्नों में से एक बना लिया। अकबर ने उन्हें ‘मियांश् की उपाधि दी। अन्य सभी विकल्प सही हैं। तानसेन ने गणेश, शिव, पार्वती एवं राम पर कई ध्रुपद रचनाएं की। उन्होंने अपने संरक्षक गीतों की रचना भी की। कल्पद्रुम उनके ध्रुपद के 300 संकलन हैं जो गौहर बानी में थे। तानसेन ने अपने पसंदीदा रागों - मुल्तानी, भैरवी एवं टोडी में रचना की। उन्होंने रात्रि राग दरबारी कान्हरा, सुबह राग मियां की टोडी, मध्याह्न राग, मियां की सारंग, मौसमी राग मियां की मल्हार का आविष्कार किया। उनके वंशज एवं शिष्य सेनिया कहलाते हैं।


77.इनमें से किस मुग़ल सम्राट ने सचित्र पांडुलिपियों से ध्यान हटाकर चित्राधार (एलबम) और वैयक्तिक रूपचित्रों पर अधिक जोर दिया

  1. हुमायूँ
  2. अकबर
  3. जहाँगीर
  4. शाहजहाँ

उत्तर (सी) एक कठिन प्रश्न! अकबर के शासनकाल के दौरान, शाही दरबार सांस्कृतिक उत्कृष्टता के केंद्र के रूप में भी उभरा। अकबर ने अपने पिता से विरासत में मिले पुस्तकालय एवं शाही एटलियर का विस्तार किया। 1560 एवं 1566 के बीच द टुटिनामा (‘टेल्स ऑफ ए पैरट’) चित्रित किया गया था। जैसे-जैसे मुगल-व्युत्पन्न पेंटिंग हिंदू राज्यों में फैली, इसके सचित्र ग्रंथों में रामायण एवं महाभारत सहित हिंदू महाकाव्य शामिल हो गए। यह जहाँगीर के शासनकाल में बदल गया, जिसके पास एक कलात्मक झुकाव था। ब्रशवर्क महीन हो गया एवं रंग हल्का हो गया। जहाँगीर यूरोपीय चित्रकला से गहरा प्रभावित था। अपने शासनकाल के दौरान वह अंग्रेजी क्राउन के सीधे संपर्क में आए एवं उन्हें तेल चित्रों के उपहार भेजे गए, जिसमें राजा एवं रानी के चित्र शामिल थे। उन्होंने अपने शाही एटलियर को पारंपरिक लघुचित्रों में प्रयुक्त चपटा बहुस्तरीय शैली के विपरीत, यूरोपीय कलाकारों के पक्ष में एकल दृष्टिकोण अपनाने के लिए प्रोत्साहित किया। उन्होंने विशेष रूप से अपने स्वयं के जीवन की घटनाओं, व्यक्तिगत चित्रों एवं पक्षियों, फूलों एवं जानवरों के अध्ययन को चित्रित करने वाले चित्रों को प्रोत्साहित किया।


78.निम्नलिखित में से कौन-सा नैशनल पार्क पूर्णतया शीतोष्ण अल्पाइन कटिबंध में स्थित है?

  1. मानस नैशनल पार्क
  2. नामदफा नैशनल पार्क
  3. नेओरा घाटी नैशनल पार्क
  4. फूलों की घाटी नैशनल पार्क

उत्तर (बी) भारत में अल्पाइन जलवायु क्षेत्र हिमालय पर्वतों में पाया जाता है, जहाँ ऊँचाई में प्रत्येक 100 मीटर की वृद्धि पर तापमान 0.6° सेल्सियस तक गिर जाता है एवं यहाँ पर तलहटी में लगभग उष्णकटिबंधीय जलवायु से लेकर ऊपर बर्फ के उस पार टुंड्रा प्रकार की विभिन्न् जलवायु पाई जाती है। जम्मू एवं कश्मीर, हिमाचल प्रदेश, उत्तराखंड, अरुणाचल प्रदेश, पश्चिम बंगाल के उत्तरी जिले एवं सिक्किम इस तरह के मौसम का अनुभव करते हैं। मानस राष्ट्रीय उद्यान असम में है। नाम्दाफा राष्ट्रीय उद्यान अरुणाचल प्रदेश में है। नीरा वैली नेशनल पार्क पश्चिम बंगाल में कालिम्पोंग जिले में है एवं फ्लावर वैली नेशनल पार्क उत्तराखंड में है। The Valley of Flowers, covering about 87.5 sq.km, is very heterogeneous geographically, varying from the temperate zone (2400-3000m) to sub-alpine forests (3000-3300m), to the alpine zone (3300-3500m). But it lies completely in the temperate alpine zone.


79.अटल नवप्रवर्तन (इनोवेशन) मिशन किसके अधीन स्थापित किया गया है?

  1. विज्ञान और प्रौद्योगिकी विभाग
  2. श्रम एवं रोजगार मंत्रालय
  3. नीति (NITI) आयोग
  4. कौशल विकास एवं उद्यमिता मंत्रालय

उत्तर (सी) अटल नवप्रर्वतन (इनोवेशन) मिशन भारत भर में नवाचार एवं उद्यमशीलता को बढ़ावा देने के लिए नीति आयोग द्वारा स्थापित एक प्रमुख पहल है। स्कूल, विश्वविद्यालय, अनुसंधान संस्थानों एवं उद्योग स्तरों पर देश भर में नवाचार एवं उद्यमशीलता के पारिस्थितिकी तंत्र को बनाने एवं बढ़ावा देने के उद्देश्य से इसका निर्माण किया गया है।


80.जून की 21वीं तारीख को सूर्य

  1. उत्तरधु्रवीय वृत्त पर क्षितिज के नीचे नहीं डूबता
  2. दक्षिणधु्रवीय वृत्त पर क्षितिज के नीचे नहीं डूबता है।
  3. मध्यान्ह में भूमध्यरेखा पर ऊर्ध्वाधर रूप व्योमस्थ चमकता है।
  4. मकर रेखा पर ऊर्ध्वाधर रूप से व्योमस्थ चमकता है।

उत्तर (ए) 21 जून या जून संक्रांति पर, अर्द्ध-रात्रि का सूर्य (मौसम अनुसार) रात भर, आर्कटिक सर्कल के बिल्कुल दक्षिण से उत्तरी ध्रुव तक दिखाई देता है। उस समय ग्रह के दूसरी तरफ, अंटार्कटिक सर्कल के दक्षिण में ‘पोलर नाइट’ होती है, जिसका अर्थ है जून संक्रांति पर यहाँ सूर्योदय नही होता है। संदर्भ छवि यहां है।


download scanned papers here - Paper 1 Paper II


81.निम्नलिखित कथनों पर विचार कीजिए :

  1. कृषि मृदाएँ पर्यावरण में नाइट्रोजन के ऑक्साइड निर्मुक्त करती हैं।
  2. मवेशी पर्यावरण में अमोनिया निर्मुक्त करते हैं।
  3. कुक्कुट उद्योग पर्यावरण में अभिक्रियाशील नाइट्रोजन यौगिक निर्मुक्त करते हैं।
  1. उपर्युक्त में से कौन-सा/से कथन सही है/हैं?
  2. केवल 1 और 3
  3. केवल 2 और 3
  4. केवल 3
  5. 1, 2 और 3

उत्तर (डी) कथन 2 निश्चित रूप से सही है। अमोनिया का 80 फीसदी उत्पादन मवेशीयों से होता हैं, हालांकि उनकी वार्षिक विकास दर स्थिर आबादी के कारण 1 फीसदी है। भारत विश्व स्तर पर अमोनिया उत्सर्जन का सबसे बड़ा स्रोत है, जो एनओएक्स उत्सर्जन का लगभग दोगुना है। तो, विकल्प (ए) सही नही है। अब कथन 3 को जांचें। पोल्ट्री उद्योग पर्यावरण में प्रतिक्रियाशील नाइट्रोजन यौगिकों को छोड़ता है। तो, 3 सही है। अब कथन 1 की जाँच करें। N2O उत्सर्जन में, 2010 में, भारत से, कृषि मिट्टी सर्वाधिक 70 फीसदी योगदान करती है, इसके बाद अपशिष्ट जल (12 प्रतिशत) एवं आवासीय एवं वाणिज्यिक गतिविधियाँ (6 प्रतिशत) होती हैं। भारतीय कृषि मे, 2002 से, N2O , मीथेन को पीछे छोड़ दूसरा सबसे बड़े ग्रीनहाउस गैस (जीएचजी) बन गया है। इसलिए सभी कथन सही हैं। तो, विकल्प (डी) सही है।


82.अलियार, इसापुर और कंग्साबती जैसे ज्ञात स्थानों में क्या समानता है?

  1. हाल ही में खोजे गए यूरेनियम निक्षेप
  2. उष्णकटिबंधीय वर्षावान
  3. भूमिगत गुफा तंत्र
  4. जल भंडार

उत्तर (डी) ये सभी दक्षिण भारत में पाए जाने वाले जलाशय हैं। जो पानी की लगातार गिरती मात्रा के कारण खबरों में थे।


83.सार्वजनिक परिवहन में बसों के लिए ईंधन के रूप में हाइड्रोजन संवर्धित CNG (H-CNG) का इस्तेमाल करने के प्रस्तावों के संदर्भ में, निम्नलिखित कथनों पर विचार कीजिए :

  1. H-CNG के इस्तेमाल का मुख्य लाभ कार्बन मोनोक्साइड के उत्सर्जनों का विलोपन है।
  2. ईंधन के रूप में H-CNG कार्बन डाइऑक्साइड और हइड्रोकार्बन उत्सर्जनों को कम करती है
  3. बसों के लिए ईंधन के रूप में CNG के साथ हइड्रोजन को आयतन के आधार पर पाँचवें हिस्से तक मिलाया जा सकता है।
  4. CNG की अपेक्षा H-CNG ईंधन को कम खर्चीला बनाती है।

उपर्युक्त में से कौन-सा/से कथन सही है/हैं?

  1. केवल 1
  2. केवल 2 और 3
  3. केवल 4
  4. 1, 2, 3 और 4

उत्तर (बी) कथन 1 सही नही है, क्योंकि ‘एचसीएनजी’कार्बन मोनोऑक्साइड उत्सर्जन को ‘समाप्त’नहीं करता है। तो, विकल्प (ए) एवं (डी) सही नही हैं। कथन 2 को देखें। यह सही है। इसलिए, हमारे पास केवल विकल्प (बी) बचा है। यही हमारा उत्तर है। संदर्भ छवि यहां है।


84.मेघाच्छादित रात में ओस की बूंदे क्यों नहीं बनती?

  1. भूपृष्ठ से निर्मुक्त विकिरण को बादल अवशोषित कर लेते हैं।
  2. पृथ्वी के विकिरण को बादल वापस परावर्तित कर देते हैं।
  3. मेघाच्छादित रातों में भूपृष्ठ का तापमान कम होता है।
  4. बादल बहते हुए पवन को भूमितल की ओर विक्षेपित कर देते हैं।

उत्तर (बी) जलवाष्प के संघनन के कारण ओस की बूंदें बनती हैं। हमारे आसपास की हवा में जल वाष्प होता है जिसे नमी या आर्द्रता के रूप में जाना जाता है। ठंडी हवा की तुलना में गर्म हवा में अधिक नमी होती है। रात के दौरान जब गर्म हवा किसी ठंडी सतह के संपर्क में आती है, तो उसमें मौजूद जल वाष्प बूंदों के रूप में ठंडी सतह पर संघनित हो जाता है। पानी की इन छोटी बूंदों को ओस की बूंदें कहा जाता है। जब आकाश साफ होता है एवं बादल कम होते हैं तो ओस का निर्माण अधिक होता है। जब आकाश साफ होता है एवं रात में पेड़ एवं पौधे ठंडे होते हैं, तो पानी का वाष्पीकरण अधिक होता है एवं इसलिए अधिक ओस बनता है। लेकिन जब वातावरण में बादल होता है, तो पेड़ एवं पौधे रात में ठीक से ठंडे नहीं हो पाते हैं (बादल रेडिएशन को परावर्तित कर देते हैं) एवं इसलिए ओस कम बनती है। संदर्भ छवि यहां है।


85.निम्नलिखित कथनों पर विचार कीजिए :

  1. भारत के संविधान के 44वें संशोधन द्वारा लाए गए एक अनुच्छेद ने प्रधानमंत्री के निर्वाचन को न्यायिक पुनर्विलोकन के परे कर दिया।
  2. भारत संविधान के 99वें संशोधन को भारत के उच्चतम न्यायालय ने अभिखंडित कर दिया क्योंकि यह न्यायपालिका की स्वतंत्रता का अतिक्रमण करता था।

उपर्युक्त में से कौन-सा/से कथन सही है/हैं?

  1. केवल 1
  2. केवल 2
  3. 1 और 2 दोनों
  4. न तो 1, न ही 2

उत्तर (बी) संविधान (चालीसवां संशोधन) अधिनियम, 1978 ने वास्तव में 1976 में आपातकाल के दौरान लाये गए 42 वें संशोधन संशोधन के कई कई बदलावों को निरस्त किया। अतः, कथन 1 सही नही है। कथन 2 सही है। यह राष्ट्रीय न्यायिक नियुक्ति आयोग (एनजेएसी) के निर्माण को संदर्भित करता है जिसे 2015 में उच्चतम न्यायालय द्वारा हटा दिया गया था।


86.निम्नलिखित कथनों पर विचार कीजिए :

  1. न्यायाधीश (जाँच) अधिनियम, 1968 के अनुसार भारत के उच्चतम न्यायालय के किसी न्यायाधीश पर महाभियोग चलाने के प्रस्ताव को लोक सभा के अध्यक्ष द्वारा अस्वीकार नहीं किया जा सकता।
  2. भारत का संविधान यह परिभाषित करता है और ब्यौरे देता है कि क्या-क्या भारत के उच्चतम न्यायालय के न्यायाधीशों की ’अक्षमता और सिद्ध कदाचार’ को गठित करते हैं।
  3. भारत के उच्चतम न्यायालय के न्यायाधीशों के महाभियोग की प्रक्रिया के ब्यौरे न्यायाधीश (जाँच) अधिनियम, 1968 में दिए गए हैं।
  4. यदि किसी न्यायाधीश के महाभियोग के प्रस्ताव को मतदान हेतु लिया जाता है, तो विधि द्वारा अपेक्षित है कि यह प्रस्ताव संसद के प्रत्येक सदन द्वारा समर्थित हो और उस सदन की कुल सदस्य संख्या के बहुमत द्वारा तथा संसद के उस सदन के कुल उपस्थित और मत देने वाले सदस्यों के कम-से-कम दो-तिहाई द्वारा समर्थित हो।

उपर्युक्त में से कौन-सा/से कथन सही है/हैं?

  1. केवल 1 और 2
  2. केवल 3
  3. केवल 3 और 4
  4. 1, 3, और 4

उत्तर (सी) कथन 1 सही नही है क्योंकि अध्यक्ष प्रस्ताव को स्वीकार कर सकता है, या इसे स्वीकार करने से इंकार कर सकता है। तो विकल्प (ए) एवं (डी) खारिज हो जाते हैं। अब कथन 4 की जाँच करें। यह वास्तव में सही है। संविधान में कार्यपालिका कार्रवाई से न्यायपालिका की स्वतंत्रता सुनिश्चित करने के उपाय हैं। इससे न्यायाधीशों को बिना किसी अनिश्चितता के स्वतंत्र एवं निष्पक्ष तरीके से न्यायिक निर्णय देने में मदद मिलती है। संविधान न्यायाधीशों द्वारा दुर्व्यवहार के खिलाफ जाँच भी प्रदान करता है। यह बताता है कि प्रत्येक सदन में दो तिहाई समर्थन वाले संसद में एक न्यायाधीश को केवल एक प्रस्ताव के माध्यम से हटाया जा सकता है। इस प्रक्रिया को न्यायाधीश (पूछताछ) अधिनियम, 1968 में रखा गया है। इस प्रस्ताव के लिए उस सदन के उपस्थित एवं मतदान के दो-तिहाई बहुमत एवं उस सदन की कुल सदस्यता का पूर्ण बहुमत होना आवश्यक है।


87.किस प्रधानमंत्री के कार्यकाल के दौरान भारत के संविधान में नौवीं अनुसूची को पुरःस्थापित किया गया था?

  1. जवाहरलाल नेहरू
  2. लाल बहादुर शास्त्री
  3. इंदिरा गाँधी
  4. मोरारजी देसाई

उत्तर (ए) नौवीं अनुसूची भारतीय संविधान में पहली बार संशोधन का उत्पाद थी। यह नेहरू सरकार द्वारा 10 मई 1951 को न्यायिक निर्णयों एवं विशेष रूप से मौलिक अधिकारों के अध्याय के बारे में घोषणाओं को संबोधित करने के लिए पेश किया गया था। इन न्यायिक घोषणाओं के परिणामस्वरूप, सरकार आशंकित थी कि संपूर्ण कृषि सुधार कार्यक्रम खतरे में पड़ जाएंगे। अनुच्छेद 31-बी को प्रथम संवैधानिक (संशोधन) अधिनियम 1951 द्वारा डाला गया था जिसमें कहा गया था कि अनुच्छेद 31-ए में निहित प्रावधानों की व्यापकता के बिना, नौवीं अनुसूची में निर्दिष्ट अधिनियमों एवं विनियमों में से कोई भी एवं इसके प्रावधानों में से कोई भी शामिल नहीं है। इसे शून्य माना जाएगा, इस प्रकार भारत के संविधान के अनुच्छेद 31-बी ने यह सुनिश्चित किया कि नौवीं अनुसूची के किसी भी कानून को अदालतों में चुनौती नहीं दी जा सकती है एवं सरकार भूमि एवं कृषि कानूनों में सुधार करके सामाजिक इंजीनियरिंग के अपने कार्यक्रम को तर्कसंगत बना सकती है। दूसरे शब्दों में, नौवीं अनुसूची के तहत कानून न्यायिक समीक्षा के दायरे से बाहर हैं, भले ही वे संविधान के भाग 3 के तहत मौलिक अधिकारों का उल्लंघन करते हैं। एक ओर अनुच्छेद 31-बी के तहत विधायिका को काफी शक्ति दी गई वहीं दूसरी ओर न्यायपालिका की शक्ति को कम किया गया है।


88.निम्नलिखित कथनों पर विचार कीजिए :

  1. भारत सरकार द्वारा कोयला क्षेत्र का राष्ट्रीयकरण इंदिरा गाँधी के कार्यकाल में किया गया था।
  2. वर्तमान में, कोयला खंडो का आबंटन लॉटरी के आधार पर किया जाता है।
  3. भारत हाल के समय तक घरेलू आपूर्ति की कमी को पूरा करने के लिए कोयले का आयात करता था, किन्तु अब भारत कोयला उत्पादन में आत्मनिर्भर है।

उपर्युक्त में से कौन-सा/से कथन सही है/हैं?

  1. केवल 1
  2. केवल 2 और 3
  3. केवल 3
  4. 1, 2 और 3

उत्तर (ए) कथन 1 वास्तव में सही है, एवं इंदिरा गांधी की सरकार ने कोयला क्षेत्र का राष्ट्रीयकरण किया है। तो, विकल्प (बी) एवं (सी) सही नही हैं। अब कथन 2 की जाँच करें। यह सही नही है क्योंकि आवंटन प्रतिस्पर्धी बोली प्रक्रिया के माध्यम से किया जाता है। इसलिए, उत्तर (ए) है।


89.निम्नलिखित कथनों पर विचार कीजिए :

  1. संसद (निरर्हता निर्वारण) अधिनियम, 1959 कई पदों को ’लाभ का पद’ के आधार पर निरर्हता से छूट देता है।
  2. उपर्युक्त अधिनियम पाँच बार संशोधित किया गया था।
  3. शब्द ’लाभ का पद’ भारत को संविधान में भली-भाँति परिभाषित किया गया है।

उपर्युक्त में से कौन-सा/से कथन सही है/हैं?

  1. केवल 1 और 2
  2. केवल 3
  3. केवल 2 और 3
  4. 1, 2 और 3

उत्तर (ए) कथन 3 रूचिकर है - किंतु सही नही है! अतः (ए) को छोड़कर सभी विकल्प खारिज हो जाते हैं। शब्द ‘लाभ का पद’ भारतीय संविधान में स्पष्ट रूप से परिभाषित नहीं किया गया है। इसीलिए संसद (निरर्हता) अधिनियम, 1959 (स्पष्टता लाने के लिए) लाया गया था। तो, ‘लाभ के पद’ की अवधारणा क्या है? सांसद एवं विधायक, विधायिका के सदस्यों के रूप में, सरकार को चलाने के लिए जिम्मेदार होते हैं। उन्हें लाभ के पद पर नही होना चाहिए का सार यह है कि अगर विधायक सरकार में होते हुए ‘लाभ का पद’ रखते हैं, तो वे सरकारी प्रभाव के लिए अतिसंवेदनशील हो सकते हैं, एवं जो उनके संवैधानिक जनादेश का निष्पक्ष रूप से निर्वहन करने में बाधा बन सकता हैं। ‘ऑफिस ऑफ प्रॉफिट’ क्या है? कानून स्पष्ट रूप से परिभाषित नहीं करता है कि ‘लाभ का कार्यालय’ क्या है, लेकिन परिभाषा विभिन्न अदालतों के निर्णयों में की गई व्याख्याओं के साथ वर्षों में विकसित हुई है। ‘ऑफिस ऑफ प्रॉफिट’ को एक ऐसी स्थिति के रूप में व्याख्यायित किया गया है जो ऑफिस-होल्डर को कुछ वित्तीय लाभ, या लाभ या लाभ प्रदान करती है। इस तरह के लाभ की मात्रा सारहीन है। 1964 में, सुप्रीम कोर्ट ने फैसला दिया कि यह निर्धारित करना कि क्या कोई व्यक्ति लाभ का पद रखता है, नियुक्ति का निर्धारण है। इस निर्धारण में कई कारकों पर विचार किया जाता है, जैसे - (1) क्या सरकार नियुक्ति प्राधिकारी है, (2) क्या सरकार के पास नियुक्ति को समाप्त करने की शक्ति है, (3) क्या सरकार पारिश्रमिक निर्धारित करती है, (4) पारिश्रमिक का स्रोत क्या है, एवं (5) पद के साथ प्राप्त होने वाली शक्तियाँ क्या है। संवैधानिक स्थिति क्या है? संविधान के अनुच्छेद 102 (1) एवं अनुच्छेद 191 (1) के प्रावधानों के तहत, एक सांसद या एक विधायक (या एक एमएलसी) को केंद्र या राज्य सरकार के तहत लाभ का कोई भी कार्यालय रखने पर रोक है। लेख स्पष्ट करते हैं कि ‘किसी व्यक्ति को केवल भारत सरकार या किसी राज्य की सरकार के अधीन लाभ का पद रखने योग्य नहीं समझा जाएगा क्योंकि वह मंत्री है’। संविधान निर्दिष्ट करता है कि मुख्यमंत्री सहित मंत्रियों की संख्या विधानसभा के सदस्यों की कुल संख्या के 15 फीसदी (दिल्ली के मामले में 10 फीसदी, जो कि विधायिका के साथ एक केंद्र शासित प्रदेश है) के भीतर होना चाहिए।


90.भारत के संविधान की किस अनुसूची के अधीन जनजातीय भूमि का, खनन के लिए निजी पक्षकारों को अंतरण अकृत और शून्य घोषित किया जा सकता है?

  1. तीसरी अनुसूची
  2. पाँचवी अनुसूची
  3. नौवीं अनुसूची
  4. बारहवीं अनुसूची

उत्तर (बी) संविधान के अनुच्छेद 244 (1) में, अनुसूचित क्षेत्रों का अर्थ ऐसे क्षेत्रों से है, जहां राष्ट्रपति आदेश दे सके कि वे अनुसूचित क्षेत्र हैं। अनुसूचित क्षेत्रों को संविधान के अनुच्छेद 244 की 5 वीं अनुसूची के तहत, जनजातीय हितों की रक्षा के लिए, स्थापित किया गया हैं।


91.हाल ही में हमारे देश में हिमालयी बिच्छू-बूटी (जिरार्डीनिया डाइवर्सीफोलिया) के महत्व के बारे में बढ़ती हुई जागरूकता थी, क्योंकि यह पाया गया है कि

  1. यह प्रति-मलेरिया औषध का संधारणीय स्त्रोत है
  2. यह जैव डीज़ल का संधारणीय स्त्रोत है
  3. यह कागज उद्योग के लिए लुगदी का संधारणीय स्त्रोत है।
  4. यह वस्त्रतंतु का संधारणीय स्त्रोत है।

उत्तर (डी) गिरार्डिनिया डायवर्सीफोलिया, जिसे आमतौर पर हिमालयन नेटल के रूप में जाना जाता है, हिमालय की तलहटी में बहुतायत से पाई जाने वाली एक प्राकृतिक जंगली झाड़ी है। सेंटर फॉर सस्टेनेबल फैशन के अनुसार, हिमालयन नेटल सबसे लंबा फाइबर है एवं लिनन की तुलना में अधिक महीन, मजबूत एवं अधिक लोचदार माना जाता है। कपड़ों के लिए नेटल फाइबर का उपयोग करने की यह नई प्रवृत्ति सिंथेटिक फाइबर से बनने वाले कपड़े से होने वाली पर्यावरणीय क्षति के प्रति चिंता से प्रेरित है।


92.निम्नलिखित में से किसका/किनका मापन/आकलन करने के लिए उपग्रह चित्रों/सुदूर संवेदी आँकड़ों का इस्तेमाल किया जाता है?

  1. किसी विशेष स्थान की वनस्पति में पर्णहरित का अंश
  2. किसी विशेष स्थान के धान के क्षेत्रों से ग्रीनहाउस गैस का उत्सर्जन
  3. किसी विशेष स्थान का भूपृष्ठ तापमान

नीचे दिए गए कूट का प्रयोग कर सही उत्तर चुनिए।

  1. केवल 1
  2. केवल 2 और 3
  3. केवल 3
  4. 1, 2 और 3

उत्तर (डी) ये सभी अनुप्रयोग वर्तमान पीढ़ी के उपग्रहों का उपयोग करके संभव हैं।


93.निम्नलिखित राज्यों पर विचार कीजिए :

  1. छत्तीसगढ़
  2. मध्यप्रदेश
  3. महाराष्ट्र
  4. ओडिशा

उपर्युक्त राज्यों के संदर्भ में, राज्य के कुल क्षेत्रफल की तुलना में वन आच्छादन की प्रतिशतता के आधार पर निम्नलिखित में से कौन-सा सही आरोही अनुक्रम है?

  1. 2-3-1-4
  2. 2-3-4-1
  3. 3-2-4-1
  4. 3-2-1-4

उत्तर (सी) 1. छत्तीसगढ़ - 41.09:, 2. मध्य प्रदेश - 25.11:, 3. महाराष्ट्र - 16.47:, 4. ओडिशा - 32.98:। इस प्रकार, ऑर्डर 3-2-4-1 होगा।


94.’मेथैन हाइड्रेट’ के निप के बारे में, निम्नलिखित में से कौन-से कथन सही हैं?

  1. भूमंडलीय तापन के कारण इन निक्षेपों से मेथैन गैस का निर्मुक्त होना प्रेरित हो सकता है।
  2. ’मेथैन हाइड्रेट’ के विशाल निक्षेप उत्तरध्रुवीय टुंड्रा में तथा समुद्र अधस्तल के नीचे पाए जाते हैं।
  3. वायुमंडल के अंदर मेथैन एक या दो दशक के बाद कार्बन डइऑक्साइड में ऑक्सीकृत हो जाता है।

नीचे दिए गए कूट का प्रयोग कर सही उत्तर चुनिए।

  1. केवल 1 और 2
  2. केवल 2 और 3
  3. केवल 1 और 3
  4. 1, 2 और 3

उत्तर (डी) मीथेन वायुमंडल में अपेक्षाकृत अल्पकालिक अस्तित्व की होती है। एक या दो दशक के भीतर मीथेन का एक अणु का पानी एवं कार्बन डाइऑक्साइड में ऑक्सीकरण हो जाता है। कथन (3) सही है। अतः (ए) उत्तर नहीं हो सकता। मीथेन हाइड्रेट बर्फ के एक पिंजरे की तरह जाली है जिसके अंदर प्राकृतिक गैस के मुख्य घटक मीथेन के अणु फंसे हुए होते हैं। यदि मीथेन हाइड्रेट या तो गर्म या अवसादग्रस्त होता है, तो यह वापस पानी एवं प्राकृतिक गैस में बदल जाता है। हाइड्रेट रिजर्व कई सौ मीटर मोटी हो सकती है एवं आम तौर पर दो प्रकार की सेटिंग्स में होती है- आर्कटिक परमाफ्रॉस्ट, एवं समुद्र तल के नीचे। कथन (2) भी सही है। जलवायु उष्मीकरण, हालांकि, हाइड्रेट्स के अस्थिर होने का कारण बन सकता है। मीथेन, जो कि एक शक्तिशाली ग्रीनहाउस गैस, वायुमंडल में बिना उपयोग के पहुँच सकती है एवं जलवायु परिवर्तन को तेज भी कर सकती है। कथन (3) भी सही है।


95.निम्नलिखित पर विचार कीजिए :

  1. कार्बन मोनोक्साइड
  2. मेथैन
  3. ओज़ोन
  4. सल्फर डाइऑक्साइड

फसल/जैव मात्रा के अवशेषों के दहन के कारण वायुमंडल में उपर्युक्त में से कौन-से निमुक्त होते हैं?

  1. केवल 1 और 2
  2. केवल 2, 3 और 4
  3. केवल 1 और 4
  4. 1, 2, 3 और 4

उत्तर (डी) फसल/बायोमास अवशेषों को प्रबंधन के लिए जलाना एक सबसे आसान एवं किफायती विकल्प है। जागरूकता की कमी या उपयुक्त प्रौद्योगिकियों की अनुपलब्धता के कारण यह प्रथा आम तौर पर सभी जगह है। फसल के अवशेषों को जलाने से न केवल वायुमंडलीय गुणवत्ता का ह्वास होता है बल्कि यह जलवायु एवं मानव स्वास्थ्य को प्रभावित करती है। फसल अवशेष एवं बायोमास जलाना (जंगल की आग) को कार्बन डाइऑक्साइड (सीओ2), कार्बन मोनोऑक्साइड (सीओ), मीथेन (सीएच4), वाष्पशील कार्बनिक यौगिकों (वीओसी), नाइट्रोजन ऑक्साइड, गैर-मीथेन हाइड्रोकार्बन (एनएमएचसी), ओजोन (ओ 3) एवं एरोसोल आदि का एक प्रमुख स्रोत माना जाता है। संदर्भ छवि यहां है।


96.निम्नलिखित युग्मों पर विचार कीजिए :

उपर्युक्त में से कौन-से युग्म सही सुमेलित हैं?

  1. केवल 1, 2 और 4
  2. केवल 1, 3 और 4
  3. केवल 2 और 5
  4. 1, 2, 3, 4 और 5

उत्तर (बी) सीरिया कहीं से भी लाल सागर को छूता तक नही है। तो 5 सही नही है, एवं इसलिए विकल्प (सी) एवं (डी) सही नही हैं। कैस्पियन सागर - कजाकिस्तान सही है। तो उत्तर केवल (बी) हो सकता है। इसके अलावा, क्रोएशिया भूमध्य सागर की सीमा पर है, न कि काला सागर की। एड्रियाटिक सागर - अल्बानिया सही है। इसलिए विकल्प (ब) सही है। संदर्भ छवि यहां है।


97.निम्नलिखित में से कौन-सा देश पिछले पाँच वर्षों के दौरान विश्व में चावल का सबसे बड़ा निर्यातक रहा है?

  1. चीन
  2. भारत
  3. म्यांमार
  4. वियतनाम

उत्तर (बी) भारत पिछले पांच वर्षों में दुनिया का सबसे बड़ा चावल निर्यातक बन गया है। लेकिन चालू वर्ष में चावल के निर्यात में गिरावट आई है क्योंकि कीटनाशकों के अत्यधिक उपयोग से भारत में चावल की गुणवत्ता खराब हो रही है। लेकिन अब शायद इन स्थितियों में सुधार हो सकता है।


98.निम्नलिखित युग्मों पर विचार कीजिए :

उपर्युक्त में से कौन-से युग्म सही सुमेलित हैं?

  1. 1, 2 और 4
  2. 1, 3 और 4
  3. 2 और 5
  4. 3 और 5

उत्तर (सी) यमुना नदी का उद्गम यमुनोत्री ग्लेशियर से हुआ है। बंदरपूँछ एक मैसिफ है, ग्लेशियर नही। अतः, 1 सही नही है, एवं उत्तर (सी) या (डी) हो सकता है। चिनाब नदी बारा शिग्री ग्लेशियर से निकलती है। अतः उत्तर (सी) है। मंदाकिनी नदी चोराबाड़ी ग्लेशियर से निकलती है एवं नुब्रा नदी रिमो ग्लेशियर से निकलती है।


99.भारत में कार्बोफ्यूरेन, मेथिल पैराथियॉन, फोरेट और ट्राइऐजोफॉस के इस्तेमाल को आशंका से देखा जाता है ये रसायन किस रूप में इस्तेमाल किए जाते हैं?

  1. कृषि में पीड़कनाशी
  2. संसाधित खाद्यों में परिरक्षक
  3. फल-पक्कन कारक
  4. प्रसाधन सामग्री में नमी बनाए रखने वाले कारक

उत्तर (ए) केरल में जैविक खेती को बढ़ावा देने के लिए, राज्य के कृषि विभाग ने 7 मई, 2015 को कीटनाशकों की दो श्रेणियों के उपयोग पर प्रतिबंध लगाने का आदेश दिया था। यह आदेश 10 दिनों के भीतर लागू किया जाना था। इस अवधि के दौरान केरल कृषि विश्वविद्यालय को प्रतिबंधित कीटनाशकों के विकल्प प्रदान करने के लिए कहा गया था, जिसमें कार्बोफ्यूरान, फोराट, मिथाइल पैराथियोन, मोनोक्रोटोफोस, मिथाइल डेमेथॉन, प्रोफेनोफोस एवं ट्रायजोफोस शामिल हैं।


100.निम्नलिखित कथनों पर विचार कीजिए :

  1. रामसर सम्मेलन के अनुसार, भारत के राज्यक्षेत्र में सभी आर्द्र भूमियों को बचाना और संरक्षित रखना भारत सरकार के लिए अधिदेशात्मक है।
  2. आर्द्र भूमि (संरक्षण और प्रबंधन) नियम, 2010, भारत सरकार ने रामसर सम्मेलन की संस्तुतियों के आधार पर बनाए थे।
  3. आर्द्र भूमि (संरक्षण और प्रबंधन) नियम, 2010, आर्द्र भूमियों के अपवाह क्षेत्र या जलग्रह क्षेत्रों को भी सम्मिलित करते हैं, जैसा कि प्राधिकार द्वारा निर्धारित किया गया है।

उपर्युक्त में से कौन-सा/से कथन सही है/हैं?

  1. केवल 1 और 2
  2. केवल 2 और 3
  3. केवल 3
  4. 1, 2 और 3

उत्तर (सी) वेटलैंड्स (संरक्षण और प्रबंधन) नियम, 2010 को पर्यावरण और वन मंत्रालय द्वारा वेटलैंड्स के क्षरण को रोकने, बेहतर संरक्षण एवं प्रबंधन सुनिश्चित करने के लिए तैयार किया गया है। वे रामसर सम्मेलन पर आधारित नहीं हैं। इसलिए, कथन 2 गलत है। इसलिए (ए), (सी) और (डी) सही नहीं हैं।









Questions and Detailed Solutions are being continuously updated ... refresh and check. Comment and let us know your experience, answers and solutions too!